You are on page 1of 68

Appendix A

Hints and Solutions to Exercises

Chapter 0 since the Fundamental theorem of recur-


n+2 sion is of outstanding importance even to
Exercise 0.1 The ansatz leads to ξ =
2 define such elementary things like addition
ξ n+1 +ξ n , such
√ that ξ = ξ+1, which is
√ solved
of the natural numbers, we will state and
by τ ∶= (1 + 5)/2 > 0 and σ ∶= (1 − 5)/2 =
prove it here for the connoisseur (and all
−1/τ < 0.
those who want to become one). Its first ap-
Let Fn = aτ n + bσ n with 0 = F0 =√a + b
plication will be the definition of factorials
and 1 = F1 √ = aτ + bσ = (a − b) 5/2.
in the next exercise.
Hence a = 1/ 5 = −b and consequently
τ n − σn Fundamental Theorem of Recursion. Let
Fn = √ . (This formula is named for
5 M be a set, η0 ∈ M , and ϕ ∈ M N0 ×M . Then
J. P. M. Binet.) Similarly, if Ln = aτ n + bσ n there exists exactly one mapping η ∈ M N0
√ 2 = L0 = a + b and 1 = L1 = 1 + (a −
with which fulfills the recurrence
b) 5/2, we get Ln = τ n + σ n .
For n ∈ N, we have η(0) = η0 , ∀ k ∈ N0 ∶ η(k + 1) = ϕ (k, η(k)) .
n Proof. Uniqueness. Let η, µ ∈ M N0 both
Fn+1 τ n+1 − σ n+1 τ − σ ( τ )
σ
= = n . fulfill the recurrence. Then η(0) = η0 = µ(0)
Fn τ n − σn 1 − ( στ ) and if η(k) = µ(k) for k ∈ N0 , then η(k+1) =
ϕ (k, η(k)) = ϕ (k, µ(k)) = µ(k + 1). By in-
But σ/τ = −1/τ 2 , whence ∣σ/τ ∣ < 1, such duction we get η = µ.
that Fn+1 /Fn → τ , as n → ∞. Similarly for Existence. The wanted function η ⊂ N0 × M
Ln . must have the following properties:
Remark A.1. While it is by an easy in- ∀ k ∈ N0 ∃ ηk ∈ M ∶ (k, ηk ) ∈ η , (A)
duction proof that one can show that there ∀ k ∈ N0 ∀ m 1 , m 2 ∈ M ∶
is only one mapping F ∈ NN0 0 which ful- (k, m1 ), (k, m2 ) ∈ η ⇒ m1 = m2 , (B)
fills the recurrence (0.4) and we proved (0, η0 ) ∈ η ∧ ∀ k ∈ N0 ∀ m ∈ M ∶
the existence by deducing Binet’s formula, (k, m) ∈ η ⇒ (k + 1, ϕ(k, m)) ∈ η . (C)
the question whether any such recurrence Condition (C) is satisfied, e.g., by
leads to a well-defined mathematical object N0 × M . Therefore we choose
is rather subtle and usually ignored in the
literature. Because of the latter fact and η = ⋂{µ ⊂ N0 × M ∣ µ fulfills (C)} .

A. M. Hinz et al., The Tower of Hanoi – Myths and Maths,


DOI: 10.1007/978-3-0348-0237-6, Ó Springer Basel 2013
266 Appendix A. Hints and Solutions to Exercises

We now prove (A) and (B) simultaneously such that from (b) we get
by induction on k. By definition, (0, η0 ) ∈ η. k k k! k!
( )+( )= +
Assume that (0, µ0 ) ∈ η for some M ∋ µ0 ≠ ` `+1 `!(k − `)! (` + 1)!(k − ` − 1)!
η0 . Let µ ∶= η ∖ {(0, µ0 )}. Then (0, η0 ) ∈ µ (k + 1)! k+1
and if (κ, m) ∈ µ, then (κ + 1, ϕ(κ, m)) ∈ µ = =( ).
(` + 1)!(k − `)! `+1
as well, since no element of the form (κ +
1, `) has been taken away from η. There-
Exercise 0.3 a) The case k = 0 is just
fore, µ has property (C); but then η ⊂ µ, in
for definiteness: of course, the empty set
contradiction to the definition of the latter
[0] can neither be permuted nor deranged
set.
in the ordinary sense of these words, but
For the induction step, put ηk+1 ∶= there is precisely one mapping from [0] to
ϕ(k, ηk ). If (k + 1, m) ∈ η for some M ∋ [0], and this mapping does not have a fixed
m ≠ ηk+1 , define µ ∶= η ∖ {(k + 1, m)}. Again point, such that f 0 = 1. On the other hand,
(C) is fulfilled for µ ((0, η0 ) has not been the only mapping from [1] to [1] clearly
excluded and (k + 1, m) ≠ (k + 1, ϕ(k, ηk ))), has a fixed point, whence f 1 = 0.
and we get the contradiction η ⊂ µ. ◻ Let k ∈ N and ` ∈ [k]. Any derange-
ment σ on [k] can be modified to yield a
derangement ̃ σ on [k + 1], namely by defin-
ing ̃σ = σ on [k] ∖ {`}, ̃ σ(`) = k + 1, and
Exercise 0.2 a) Induction on `. There ̃
σ(k + 1) = σ(`), i.e. by mapping ` on k + 1
is exactly one (injective) mapping from and the latter to the former image of `. This
[0] = ∅ to [k]. An injection ι from [` + 1] makes up for k ⋅ f k derangements on [k + 1].
to [k], ` ∈ [k]0 , can have k values for A derangement σ on [k − 1] can be
ι(` + 1), and ι ↾ [`] (↾ stands for “restricted manipulated as follows. Define τ ∶ [k −1] →
to”) runs through all injections from [`] [k] ∖ {`} by τ (i) = i for i < ` and τ (i) = i + 1
to [k] ∖ {ι(` + 1)}, such that there are all otherwise. This yields a derangement ̃ σ on
(k − 1)! k! [k +1] by putting ̃ σ = τ ○σ ○τ −1 on [k]∖{`},
in all k = injec-
((k − 1) − `)! (k − (` + 1))! ̃
σ(`) = k + 1, and ̃ σ (k + 1) = `. This leads to
tions from [` + 1] to [k]. another k ⋅ f k−1 derangements on [k + 1]
The statement about bijections fol- which are obviously different from the ones
lows from the special case ` = k, together constructed earlier by the fact that they
with the pigeonhole principle which ex- just switch k + 1 with some ` ∈ [k]. Con-
cludes the possibility of an injection from versely, a derangement on [k + 1] either has
[k] to a smaller set. this property or not, such that it is easy to
convince oneself that all of them are among
b) Again by the pigeonhole principle, there the k (f k +f k−1 ) constructed ones. We thus
is no subset of [k] with ` > k elements. For observed the two basic rules of counting,
k! namely not to forget any object and not to
k ≥ `, we have from (a) injective
(k − `)! count any twice.
mappings from [`] to [k]. Each element of b) To show that (0.14) follows from (0.15)
[k] is easy: clearly, x1 = 0 and for k ∈ N, we can
( ) occurs `! times as a permuted image
` add the two equations
set of one of these injections.
xk+1 = (k + 1)xk − (−1)k ,
c) For k < `, all three terms are 0. For k = `,
xk = kxk−1 + (−1)k ,
k+1 k k
we have ( ) = 1 = ( ) and ( ) = 0.
`+1 ` `+1 to obtain xk+1 + xk = (k + 1)xk + kxk−1 ,
If k > `, then k +1 ≥ `+1, k ≥ `, and k ≥ `+1, whence xk+1 = k (xk + xk−1 ).
Appendix A. Hints and Solutions to Exercises 267

As Euler admits himself, “however, it 1110 = 10112 , such that every bit of 11 is
is not as easy” to derive (0.15) from (0.14). smaller than or equal to the corresponding
Therefore, we first prove formula (0.16) as- one of 1039 and therefore (1039
11
) is odd ac-
suming (0.14) by induction, where the in- cording to (0.8).
duction step for k ∈ N reads
Exercise 0.5 Employ Fleury’s algorithm:
xk+1 = k(xk + xk−1 ) start in a vertex of odd degree, A say, avoid
using a bridge (edge) that separates the re-
k−1
(−1)` (−1)k k maining graph into two components, and
= (k + 1)! ( ∑ + )
`=0 `! (k + 1)! delete used edges. For example:
k+1
(−1)` ABCDABDAC.
= (k + 1)! ∑ .
`=0 `! Exercise 0.6 We can employ Fleury’s al-
gorithm for the graph in Figure A.1, whose
Finally, the implication “(0.16) ⇒
vertices are all even. Starting with edge a,
(0.15)” is trivial.
we follow Fleury to get the trail abcdef,
Exercise 0.4 103910 = 100000011112 and here denoted by the labels of the edges.

f c

d a

b e

Figure A.1: Muhammad’s sign manual as a graph

Exercise 0.7 We will describe the algo- n ∶= ∣G∣ for all v ∈ V (G). The length ` of the
rithm informally. To check an edge ab for queue is put to 1 and qb = 1, such that ver-
being a bridge in G, we search for a path tex b is now first in the line. Then we search
from b to a in G − ab. Starting in b, we successively for its neighbors u in G − ab
screen all neighbors of b, then neighbors of which have not yet been visited, i.e. with
neighbors and so on, but skipping those al- qu = n. If we encounter a, we stop: ab is
ready encountered. (Since we stay as close not a bridge. Otherwise we increase ` by 1
to the root b as possible, this is called a and put qu = `. After all neighbors of b have
breadth-first search (BFS)). The result is a been scanned, we decrease ` by 1 and also
tree rooted in b and spanning its connected all qu ∈ [n − 1] and restart the search for
component in G − ab. If a is not on that neighbors of the first vertex v in the queue,
tree, then ab is a bridge. that is with qv = 1, as long as there is any,
i.e. ` = ∣{u ∈ V (G) ∣ qu ∈ [n − 1]}∣ ≠ 0.
For a practical realization, we may
construct a queue of vertices to be checked Exercise 0.8 a) Coming from L T, the next
for their neighbors. Initially we define qv = three moves are µ λ µ, which can be found
268 Appendix A. Hints and Solutions to Exercises

twice in both equations (0.9) and (0.10), never outnumber the missionaries on a river
such that there are the four solutions bank.
Identifying women with cannibals
L T S R Q Z B G H X W V J K F D C P N M, and men with missionaries, it is clear that
L T S R Q Z X W V J H G B C P N M D F K, every solution of CB leads to a solution
L T S R Q Z B C P N M D F G H X W V J K, of MC, because individuals do not play a
L T S R Q P N M D C B Z X W V J H G F K. role in the latter and women can never
b) Here the triple is µλ2 , occurring outnumber men on a bank since otherwise
only in one kind in each equation, such that a woman would be without her brother.
we have the two solutions So if we solve MC, we only have to ver-
ify that the solution will also satisfy CB.
J V T S R W X Z Q P N M L K F D C B G H, We will stay with the women/men nota-
J V T S R W X H G F D C B Z Q P N M L K. tion and denote by mw a constellation of
m ∈ {0, 1, 2, 3} men and w ∈ {0, 1, 2, 3}
Exercise 0.9 Let us assume that K3,3 is women on the bank where the boat is
planar. Since it is 2-colorable, it does not present. By the jealousy condition, only
contain any triangle. Therefore 4∣∣∣K3,3 ∣∣∣ ≤ 10 of the 16 combinations are admissi-
2∥K3,3 ∥, such that with (0.11), we arrive at ble: 00, 01, 02, 03, 11, 22, 30, 31, 32, 33. Of
9 = ∥K3,3 ∥ ≤ 2(∣K3,3 ∣ − 2) = 8, a contradic- these 00 (no transfer possible), 01 (can only
tion. be reached from 33 and leads back there)
and 30 (more women than men on the other
Exercise 0.10 Hint: Reduce problem CB bank after transit) can be excluded imme-
to the missionaries and cannibals problem diately. The remaining 7 constellations lead
(MC): three missionaries and three can- to the graph in Figure A.2, whose edges
nibals want to cross the river with the are boat transfers labelled with the passen-
same boat as before; the cannibals must ger(s).

02
ww w
33 ww w mm mw
32 03 31 22
mw m
11
Figure A.2: The graph of the missionaries and cannibals problem

Note that this graph contains a loop at ver- In fact, all solutions lead to solutions
tex 22. Quite obviously, there are precisely of (CB), where now individual identities
four optimal solutions of length 11. One of increase the number of different solutions.
the solutions has been given in Latin verses For instance the transfer 33 → 02 can be
(cf. [265, p. 74]): performed with any of the three pairs of
women, whereas the reverse transfer has
Binae, sola, duae, mulier, duo, vir mulierque,
to be done with the only pair present. A
Bini, sola, duae, solus, vir cum mulier.
combinatorial analysis of all cases based
Appendix A. Hints and Solutions to Exercises 269

on Figure A.3 leads to 486 solutions alto- land in the middle of the river; cf. [265].
gether. It seems that this number has never The problem has also been employed as an
been given in the abundant literature of the example for problem representations, i.e.
problem. models, in the Artificial Intelligence liter-
Variations include higher numbers of ature; cf. [12].
couples and (remember Königsberg!) an is-

2
1 2
3 2
1 3 3 1 1 1

3 1
1 1

Figure A.3: Numbers of alternatives in the careful brothers problem

Chapter 1 Exercise 1.3 In every move precisely one


bit is switched. This yields the vertex col-
Exercise 1.1 The two pendant vertices are
oring. It can be proved by induction that
the end-vertices of a path. By the Hand-
moves of type 0 and 1 are alternating along
shaking lemma the rest of the vertices form
a shortest path from α(n) to ω (n) , obviously
a union of cycles.
starting and ending with move type 0.
Exercise 1.2 The path of length 2n − 1
from α(n) to ω (n) in Rn can be dissected Exercise 1.4 a) The state graph is con-
into the two paths from 0n to 1n and from structed by adding edges {s00, s11} with
1n = 11n−1 to ω (n) = 10n−1 of lengths βn s ∈ B n−2 to Rn , just like the red edges in
and βn−1 , respectively. Figure A.4.

000 001 011 010 110 111 101 100


Figure A.4: The graph R3 with double moves (red edges)

b) We may define the graph R ̃n from the β̃n+1 + β̃n = 3 ⋅ 2n−1 − 1. For γn ∶= 2n−1 − β̃n
previous graph by deleting all vertices of this means that γ1 = 0 and ∀ n ∈ N ∶ γn+1 =
the form s01, because they subdivide the 1 − γn , whence
new edges. It is a path graph again, whose
length is 2n − 1 + 2n−2 − 2 ⋅ 2n−2 = 3 ⋅ 2n−2 − 1. β̃n = 2
n−1
− (n even) .
c) As in (1.2), we have β̃1 = 1, ∀ n ∈ N ∶
270 Appendix A. Hints and Solutions to Exercises

Exercise 1.5 Hint: (2n −1) mod 3 = n mod Exercise 1.10 Hint: Starting at 0n , walk
2. For n = 0, C = V and ∣C∣ = 1. For along the edges of the cube by switching
n ∈ N, either 0n or 0n−1 1 is a codeword. bgk in the k-th step for k ∈ [2n − 1]. This
Consequently, the codewords are those ver- path will end in 0n−1 1, corresponding to a
tices s ∈ B n with d(s) mod 3 = 0 or 1, re- corner of the n-cube which is linked to our
spectively. From the hint it follows that for starting corner by an extra edge.
even n necessarily α(n) and ω (n) are code-
1 n Exercise 1.11 We will show that Corol-
words and that ∣C∣ = (2 + 2); for odd n lary 1.11 remains valid with g replaced by
3
there are two perfect codes, containing ei- a. We first claim that for any r ∈ N0 , (i)
1 n
ther α(n) or ω (n) and with ∣C∣ = (2 + 1). a2r = r + 1, and that (ii) ak < r + 1 for any
3 k < 2r .
Exercise 1.6 We are looking for the state We proceed by induction on r, the
s ∈ B 14 which lies on the path R14 between cases r = 0 and r = 1 being trivial. Let
114 and 014 at a distance 9999 from the for- r ≥ 2 and suppose that a2r−1 = r and that
mer. Since β14 = d(114 , s)+d(s, 014 ), we get the terms before a2r−1 are smaller than r.
2 The 2r−1 − 1 terms after the term a2r−1
d(s, 014 ) = ⌈ (214 − 1)⌉ − d(114 , s) constructed by the greedy algorithm coin-
3
= 10922 − 9999 = 923 cide with the first 2r−1 − 1 terms of the
= 000011100110112 . sequence. Indeed, if we would construct a
square, it would not contain a2r−1 because
We can now apply the Gray code au- by the induction hypothesis the terms be-
tomaton in Figure 1.4 which yields s = fore it are smaller. So a square would
00001001010110. Therefore, rings 2, 3, 5, have to appear before (or, equivalently, af-
7, and 10 are on the bar, all others are off. ter) a2r−1 , which is not possible. Hence we
Exercise 1.7 Obviously, W (P1 ) = 0, and have a sequence containing (2r−1 − 1) +
for a pendant vertex e of Pk , k > 1, we have 1 + (2r−1 − 1) = 2r − 1 terms of the form
W (Pk ) = ∑ d(e, v) + W (Pk−1 ) 1, 2, . . . , 1, r − 1, 1, 2, . . . , 1, r, 1, 2, . . . , 1, r −
v∈V (Pk ) 1, 1, 2, . . . , 1. Now we see that a2r = r+1, for
k otherwise a square would be constructed.
= ∆k−1 + W (Pk−1 ) = ( ) + W (Pk−1 ) . Moreover, the above argument also implies
2
part (ii) of the claim.
Comparing this with Exercise 0.2(c), we re-
Suppose now that k = 2r (2s + 1),
alize that W (Pk−1 ) fulfills the same recur-
where s ≥ 1. Let t ∈ N be selected such that
rence as (k3), such that W (Pk ) = (k+1 ).
3 2t < k < 2t+1 . To show that ak = r + 1 in
Exercise 1.8 Write k = 2r (2s + 1), r, s ≥ 0. this case too, we proceed by induction on
Then observe that in the binary representa- t, where the case t = 1 is straightforward.
tion (bn bn−1 . . . b1 b0 )2 of k we have b0 = b1 = So let t ≥ 2. Then by the above, ak = ak−2t .
⋯ = br−1 = 0 and br = 1. Hence ̃ gk = r. Since Since k − 2t = 2r (2s + 1) − 2t = 2r ((2s + 1) −
by Corollary 1.11, gk = r + 1, the conclusion 2t−r ) and (2s + 1) − 2t−r is odd, the induc-
follows. tion hypothesis implies ak−2t = r + 1. But
then ak = r + 1 as well.
Exercise 1.9 The recursion for q follows
As an alternative, we may prove
immediately from the binary representa-
Proposition 1.12 by using induction on l ∈
tion of k. But then gk = q(k − 1) − q(k) + 2,
N to show that a2l−1 = 1, a2l = al + 1
because the right-hand side fulfills the re-
(cf. Proposition 1.10).
cursion of the Gros sequence in Proposi-
tion 1.10. Summing ̃ gk = q(k − 1) − q(k) + 1 Exercise 1.12 Hint: Use the fact that g is
from 1 to n yields Legendre’s formula. the greedy square-free sequence and argue
Appendix A. Hints and Solutions to Exercises 271

that g is also the greedy strongly square- (i) Never place a disc onto a disc or
free sequence. base of the same metal.
Rule (o) is obvious for a shortest so-
lution path and was the only rule to be fol-
Chapter 2 lowed to get from α(n) to ω (n) in the CR.
Rule (i) will guarantee by Proposition 2.3
Exercise 2.1 Observe that for any r, s ∈
that you stay on the optimal path.
T m , r ≠ s, rT n−m ∩ sT n−m = ∅. The rest
follows from the fact that sT n−m is the set Exercise 2.6 Let bn , n ∈ N0 , be the number
of regular states with the largest m discs of legal arrangements on the source peg i in
fixed in distribution s. the optimal solution with n discs. Note that
bn is at the same time the number of legal
Exercise 2.2 We may assume that the
arrangements of discs on the goal peg j.
transfer of the tower is from peg 0 to peg 2.
Then b0 = 1 = F2 and b1 = 2 = F3 . For
Replace the induction step in the proof of
n ≥ 2, we first move discs 1 to n − 1 onto
Theorem 2.1 by the following argument.
the intermediate peg 3 − i − j, yielding bn−1
Before the first move of disc d, to peg i,
arrangements on the source peg. After disc
say, state 0n−d 0(3 − i)d−1 must be reached
n is moved to the goal peg, discs 1 to n − 2
moving only discs from [d − 1] in the min-
are moved from peg 3 − i − j to peg i, yield-
imum number of moves, which by (strong)
ing bn−2 arrangements on the source peg.
induction assumption is 2d−1 − 1.
Now prove that these two sets of arrange-
The statement about the last step fol-
ments are disjoint and observe that after
lows by symmetry, since every minimal so-
that no new arrangement on the source peg
lution from 0n to 2n is a minimal solution
is obtained. It follows that bn = bn−1 + bn−2 .
from 2n to 0n taken in inverse move order.
The argument is completed by the fact that
The statement about the disc moved
an = bn−1 .
in step k in Theorem 2.1 then follows a pos-
A second proof can be based on
teriori from another characterization of the
Proposition 2.3. By that result, bn is also
Gros sequence reflecting the corresponding
the number of admissible arrangements on
property of the Gray code, namely
peg i. Now let ν ∈ N0 . If n = 2ν, then peg i is
∀ n ∈ N0 ∶ g2n = n + 1, either empty (we only count movable discs)
n or the bottom disc is 2k + 2, k ∈ [ν]0 , lead-
∀ k ∈ [2 − 1] ∶ g2n +k = g2n −k .
ing to b2k+1 admissible arrangements on i,
This, in turn, is an easy consequence of i.e.
ν−1
Corollary 1.11. b2ν = 1 + ∑ b2k+1 . (A.1)
k=0
Exercise 2.3 Hint: Use the recursive solu-
If n = 2ν+1, then again peg i is either empty
tion of the problem and induction.
or the bottom disc is 2k + 1, k ∈ [ν + 1]0 ,
Exercise 2.4 Hint: Start with h. leading to b2k admissible arrangements on
peg i, i.e.
Exercise 2.5 Let us assume that disc n
ν
and all discs of the same parity are made
b2ν+1 = 1 + ∑ b2k . (A.2)
of gold, the others from silver. Moreover, k=0
the bases of pegs 0 and 2 should be silver,
that of peg 1 golden. Then, apart from the From (A.1) it follows that b2ν + b2ν+1 =
divine rule, of course, follow just two com- b2ν+2 , and from (A.2) we get b2ν+1 + b2ν+2 =
mandments: b2ν+3 , i.e. the recurrence relation of the Fi-
(o) Never undo the move just made. bonacci sequence is satisfied by (bn )n∈N0 .
272 Appendix A. Hints and Solutions to Exercises

(In fact, (A.1) and (A.2) correspond to cept disc 5, leads to a change of state of the
an equivalent characterization of that se- P1-automaton of Figure 2.6, such that the
quence; cf. [213, p. 5].) algorithm returns δ = 1 and µ = 239; it ends
Exercise 2.7 The proof is by induction on in state 2. Hence we know that the minimal
n. The case n = 1 is clear. Let n ∈ N. Then solution has length 239 and starts with the
disc n + 1 moves by (j − i) mod 3 only once move of disc 1 from peg 1 to peg 0. There-
in move number 2n . Before that move, disc after, one can use the statement of Propo-
d ∈ [n] is relocated in moves (2k + 1) 2d−1 , sition 2.9 to obtain the rest of the optimal
k ∈ [2n−d ]0 , by path.
(((3 − i − j) − i) ((n − d) mod 2 + 1)) mod 3 Exercise 2.10 a) Apply Algorithm 10
= ((i − j) ((n − d) mod 2 + 1)) mod 3 for i and j, i.e. on two copies if the P1-
= ((j − i) (((n + 1) − d) mod 2 + 1)) mod 3 automaton, starting one in state i, the
other in state j. Then, as long as sd = k,
from induction assumption. Similarly for
the states of the automata will just be
the moves after the largest disc’s move,
switched and 2d−1 be added to µ. As soon as
when k ∈ [2n+1−d ]0 ∖ [2n−d ]0 .
sd ≠ k, however, only one of the automata
Exercise 2.8 We assume the initial nee- will change its state and add 2d−1 to µ. This
dle to be 0, the goal to be 2. The age of can not be compensated during the rest of
the universe (in moves) can be determined the performance.
if we know the state s ∈ T 64 of the Tower b) The initial state is s = 0n 1n , and
of Brahma after the current move of disc δ. the result will depend on the parity of n.
If δ ≠ 1, this move takes place between the If either n is odd and the goal peg is 1 or
two pegs which are not occupied by disc 1, n is even and the goal is 2, then the P1-
and by Proposition 2.4 the parity of δ de- automaton changes its state with every in-
cides on the direction of the move. If δ = 1, put of an sd , such that all in all 4n −1 moves
we know that d(s, 0n ) has to be odd. Al- are necessary. Otherwise, the automaton
though we do not know the value of s1 , we stays in the same state after the inputs from
can run the recursion in (2.5) down to d = 2 the n largest discs, such that only 4n − 2n
and put `0 = 1. moves are necessary. Quite obviously, the
Disc 58, being even, necessarily moves distance of s to the perfect state 02n is just
from 0 via 2 to 1. Therefore the goal of the 2n − 1.
current move is needle 1 because disc 1 lies
Exercise 2.11 Moving one step from s to
on needle 0. Therefore, sd = (d = 59) + (d =
t, say, s and t differ in precisely one tit:
58) and application of (2.5) yields that the
sd ≠ td . But then d(s, kn ) and d(t, kn ) can
move number is 258 + 257 (one may coun-
differ by at most 1, and by Lemma 2.8 they
tercheck with equations (2.3) and (2.4)),
cannot all be pairwise equal. The rest fol-
such that the age of the universe is about
lows from Proposition 2.13 as applied to s
13.7 billion years. (Note that this is in
and t, respectively.
amazingly good agreement with the present
knowledge of cosmology! So it seems that Exercise 2.12 We may assume that j =
the Brahmins have not yet touched disc 60 1. Then, by Proposition 2.13, d(s, 0n ) +
and that the universe will survive for a d(s, 2n ) = 2n − 1. Hence s lies on the short-
while.) est path from perfect state 0n to perfect
state 2n . There are 2n such states.
Exercise 2.9 The task is to get from state
01210021 to perfect state 18 . We apply Al- Exercise 2.13 The number of moves can be
gorithm 10, starting with k = 1 correspond- obtained from Proposition 2.19 as the value
ing to the goal peg. Each of the discs, ex- of y8 , which is 182. This number can also
Appendix A. Hints and Solutions to Exercises 273

be found by applying the P1-automaton to using color 3.


the task to get from (01)4 to 28 . We can-
not use the information about the idle peg Exercise 2.16 One way to prove this
for this task directly, because we are deal- is to combine the Handshaking lemma
ing with the inverse task. However, since with Exercise 2.14. Other approaches are
disc 8 will only move once from peg 1 to to solve the recurrence ∥H30 ∥ = 0, ∥H31+n ∥ =
peg 0, the optimal solution will start with 3(∥H3n ∥ + 1), stemming from (2.10), or to
the transfer of a 7-tower from 1 to 2, which use (2.9) directly.
means that disc 1 is moved to peg 2 at the
beginning. The rest of the solution should Exercise 2.17 a) The proof is by induction
also be clear. on n, the case n = 1 being obvious. For the
induction step note first that any hamilto-
Exercise 2.14 By the definition of TH it nian path from 11+n to 21+n cannot use the
follows easily that the perfect states are of edge between 10n and 20n . But then the
degree 2 and all the other vertices are of hamiltonian path is obtained uniquely from
degree 3. induction assumption, putting together the
Exercise 2.15 We may, of course, assume unique hamiltonian paths on 1H3n , 0H3n ,
that n ∈ N. The vertices can be colored and 2H3n , respectively. As an illustration,
with colors 0, 1, and 2 according to Propo- the three hamiltonian paths between i4 and
sition 2.21. Each of the 3n−1 subgraphs in- j 4 , i ≠ j, can be seen in Figure 2.10 by delet-
duced by the three vertices s0, s1, and s2 ing the edges colored like vertex (3 − i − j)4 .
with s ∈ H3n−1 (they are isomorphic to the b) Hint: Use part (a). In Figure A.5, follow
complete graph K3 ) can obviously be to- the 27(!) letters of the English alphabet on
tally colored with colors 0, 1, and 2, re- the graph H33 .
specting the vertex colors already assigned. c) Clearly, H30 and H31 are eulerian. All the
For n > 1, the remaining edges, representing others are not even semi-eulerian, because
moves of discs 2 to n, can then be colored there are 3n −3 > 2 vertices of odd degree 3.

m o

l p

j r
k q
i s

g h t u

f b z v

e d c a & y x w

Figure A.5: Hamiltonian labelling of H33


274 Appendix A. Hints and Solutions to Exercises

Exercise 2.18 Let an ∶= ∣An ∣ and cn ∶= ∣Cn ∣. We get the following recurrence rela-
By construction, a0 = 0 and c0 = 1. More- tion for the numbers mn of matchings of
over, H3n , depending on the number `n of match-
ings of H3n with one, kn with two, and jn
an+1 = an + 2 ((n even)an + (n odd)cn ) , with three perfect states deleted:
cn+1 = cn + 2 ((n even)an + (n odd)cn ) ,
such that in particular cn+1 −an+1 = cn −an , m1+n = m3n + 3mn `2n + 3`2n kn + kn3 .
whence cn = an + 1. Therefore,

an+1 = an + 2((n even)an In turn, a matching of an H31+n with


two perfect states deleted will also always
+ (n odd)(an + 1))
be composed as in Figure A.6, which now
= 3an + 2(n odd) . has to be read with the two lower perfect
states deleted. The remaining unsymmetric
It follows that
cases are shown in Figure A.7.
n−1
k This leads to
an = 2 ∑ 3 (n − 1 − k odd)
k=0
n−1 k1+n = mn `2n + mn kn2 + 2kn `2n
k n−1−k
= ∑ 3 (1 − (−1) ) + kn3 + 2jn kn `n + jn2 kn ;
k=0

and consequently similar arguments yield two more equations


n n n obtained by interchanging m with j and k
3 − 1 3 − (−1)
an = − with ` simultaneously. The seeds of this re-
2 4
currence are m0 = 1 = l0 , k0 = 0 = j0 .
1 n
= (3 − 2 + (−1)n ) b) The recurrence for the number of respec-
4
tive perfect matchings is the same as for
and cn = 41 (3n + 2 + (−1)n ). matchings in general, except for the seeds,
A more direct proof can be based on which are m0 = k0 = j0 = 0, l0 = 1. Obvi-
the fact that the number of perfect states ously, mn = 0 = kn for all n, because the cor-
which are codewords is 2 + (−1)n . Each responding graphs are of odd order. There-
of them covers 3 vertices, while the non- fore the recurrence reduces to
perfect codewords cover 4. So we only have
to solve the equation `1+n = jn `2n + `3n ,

(2 + (−1)n ) ⋅ 3 + (cn − (2 + (−1)n ) ⋅ 4 = 3n . j1+n = jn3 + `3n .

Exercise 2.19 a) Any matching of H31+n is Since `1 = 1 = j1 , we see that `n = jn


the union of matchings of H3n or graphs ob- for all n ∈ N. So we are left with solving the
tained from H3n by deleting perfect states. recurrence
In Figure A.6 one can find all possible con-
`1 = 1, ∀ n ∈ N ∶ `1+n = 2`3n .
figurations depending on the number of
connecting edges included in the match-
It is obvious that `n = 2λn with the se-
ing, the two middle ones occurring in three
quence (λn )n∈N fulfilling λ1 = 0 and λ1+n =
symmetric versions each; the little trian-
3λn + 1. By Lemma 2.18, we get λn =
gles represent copies of matchings of H3n √ 3n
with perfect states deleted shown as un-
1
2
(3n−1 − 1), whence `n = √12 6 2 . (Note
filled dots. that λn = 13 ∥H3n−1 ∥; cf. Exercise 2.16.)
Appendix A. Hints and Solutions to Exercises 275

Figure A.6: Composing matchings for H31+n

Figure A.7: Composing matchings for a reduced H31+n

Exercise 2.20 By definition, for any two in Lemma 2.29.


vertices s̄ and t̄ of H3∞ , there is an n ∈ N0 , The second equation follows immedi-
such that s̄ = 0∞ s, t̄ = 0∞ t, s, t ∈ H3n . Hence ately from this and (2.23).
there is a path between s and t in H3n and Let an ∶= ∑ µzn (µ). Then, by (2.20),
consequently there is a (finite) path be- µ∈N
tween s̄ and t̄ in H3∞ . a0 = 0 and

Exercise 2.21 From the formula in Propo- an+1 = ∑ µzn+1 (µ)


sition 2.27 we see that ε(is) becomes min- µ∈N

imal, if d(s, in ) − d(is) gets maximal. Re- = ∑ µ (zn (µ − 2n ) + zn (µ) + zn (µ + 2n ))


mark 2.12 tells us that d(is) = 0 ⇔ s = in , µ∈N

in which case ε(i1+n ) = 2n+1 − 1. Otherwise, = ∑ (µ < 2n )µ (zn (µ − 2n ) + zn (µ))


ε(is) ≥ 2n+1 − 1 − 12 (2n − 1 − 1) = 3 ⋅ 2n−1 . µ∈N

The latter value is strictly less than 2n+1 −1 + 2n zn (0) + ∑ (2n < µ)µzn (µ − 2n )
µ∈N
for n > 1 and then attained only for ver-
tices is with d(s, in ) = 2n − 1 and d(is) = 1. = ∑ µzn (µ) + 2n ∑ zn (µ) = an + 6n ,
µ∈N µ∈Z
Since s ∈ T n is uniquely determined by the
two values d(s, in ) and d(s; k, j), there are where use has been made of (2.23).
precisely 6 vertices fulfilling this, namely Lemma 2.18 yields the third identity in
ijin−1 , i, j ∈ T, i ≠ j. Lemma 2.29.
The periphery of H31+n consists of
those vertices is for which, without loss Exercise 2.23 We first remark that by an
of generality, d(s, in ) = d(s; k, j), or easy induction argument it follows from ei-
d(is, j 1+n ) = 2n+1 − 1, i.e. all vertices which ther (2.27) or (2.28) that ∀ n ∈ N0 ∶ b(2n ) =
have maximal distance to one of the per- 1.
fect states. For n ≤ 1, center and periphery Now assume that (2.27) holds. Then
coincide. b(2) = b(1) by the previous statement and
from case n = 1 of (2.27) we have b(3) =
Exercise 2.22 By definition of zn (µ) for a b(1) + b(1) = b(1) + b(2). Let (2.28) be true
fixed n, every s ∈ T n is counted for exactly up to ν − 1 ∈ N. Since we may assume that
one µ ∈ Z, which proves the first equation 2n < 2ν < 2n+1 for some n ∈ N, we have
276 Appendix A. Hints and Solutions to Exercises

2n−1 < ν < 2n . Then z such that the triangle xzy has a 60○ an-
gle at z. Let a ∶= ∥x − y∥, b ∶= ∥x − z∥, and
b(2ν) = b(2n+1 − 2ν) + b(2ν − 2n )
c ∶= ∥z − y∥. Then, by the cosine theorem,
= b (2(2n − ν)) + b (2(ν − 2n−1 )) a2 = b2 +c2 −2bc cos(60○ ) = b2 +c2 −bc. It fol-
= b(2n − ν) + b(ν − 2n−1 ) lows that 4a2 = (b + c)2 + 3(b − c)2 ≥ (b + c)2 ,
= b(ν) . whence 2a ≥ b + c. Now d(x, y) ≤ d(x, z) +
d(z, y) = ∥x − z∥ + ∥z − y∥ ≤ 2∥x − y∥.
Similarly, For the induction step we look at the
b(2ν + 1) = b (2n+1 − (2ν + 1)) straight line section between the two points
+ b ((2ν + 1) − 2n ) x and y on σ n+1 . Starting in x let x̃ be the
first point on σ n (if any) and ỹ the last
= b (2(2n − (ν + 1)) + 1)
point on σ n before we reach y (if any). Then
+ b (2(ν − 2n−1 ) + 1) for the (at most) one segment between x̃
= b (2n − (ν + 1)) + b(2n − ν) and ỹ we can use the induction assump-
+ b(ν − 2n−1 ) + b ((ν + 1) − 2n−1 ) tion, and the remaining segments can be
treated like the case n = 0, just in a smaller
= b(ν) + b(ν + 1) ,
triangle.
where we note that the equations in (2.28) If we choose x and y to be the center
are trivially valid for ν = 0. points of two sides of the middle triangle,
Now assume that (2.28) holds. The i.e. the one to be added in the step from
case n = 0 is clear. Let (2.27) be valid up to n = 0 to n = 1, then d(x, y) = 21 = 2∥x − y∥.
n − 1 ∈ N0 and let 2n < ν < 2n+1 . If v = 2µ,
Exercise 2.25 By isomorphy, the number
then 2n−1 < µ < 2n and we get
we are looking for is equal to the number
b(ν) = b(2µ) = b(µ) of states which are at a distance of µ from
a perfect state of the TH. So the statement
= b(2n − µ) + b(µ − 2n−1 ) follows immediately from Proposition 2.16.
= b(2n+1 − ν) + b(ν − 2n ) . An alternative proof can be based on
(0.8); this was Glaisher’s argument in [111,
For ν = 2µ+1, where 2n−1 ≤ µ < 2n , we have p. 156].
b(ν) = b(2µ + 1) = b(µ) + b(µ + 1) Exercise 2.26 As b(ν) is the number of odd
= b(2n − µ) + b(µ − 2n−1 ) entries in the νth subdiagonal of AT mod 2,
all entries of which add up to the Fibonacci
+ b (2n − (µ + 1)) + b ((µ + 1) − 2n−1 )
number Fν , b(ν) is even if and only if Fν
= b (2(2n − (µ + 1)) + 1) is (cf. Figure 2.26). From the recurrence for
+ b (2(µ − 2n−1 ) + 1) the Fibonacci numbers it is easy to see that
= b(2n+1 − ν) + b(ν − 2n ) .
this is the case if and only if 3 divides ν.

Chapter 3
Exercise 2.24 We show by induction on
n ∈ N0 that the inequality is true if x and y Exercise 3.1 As the start vertex take the
lie in the union σ n of all constituting subtri- state σ = 1 . . . (n − 2) n (n − 1) ∣ ∣ from
angles of side-lengths greater than or equal Remark 3.4. Then we have seen in this
to 2−n ; it then follows for the remaining remark that it takes 2n−2 moves to get
pairs by continuity. If n = 0, the points x disc n away from peg 0. After this first
and y lie on the triangle with side-lengths 1. move of the largest disc the state is ei-
Among the corners of that triangle choose ther n − 1 ∣ n ∣ 1 . . . n − 2 = 102n−2 or
Appendix A. Hints and Solutions to Exercises 277

n − 1 ∣ 1 . . . n − 2 ∣ n = 201n−2 . If we choose i.e. the bottom of peg 2 and a recursive


t = 0n as the target state, then it is clear call is made for the task 6 3 ∣ 4 1 7 ∣ →
that disc n has to be moved (at least twice) ∣ ∣ 1 3 4 6 7 . This, in turn, is a standard
if we want to get from σ to t, and from case task which the algorithm analogously
formula (2.8) we deduce that the distance solves in 12 moves. Then disc 8 moves from
from the intermediate state to t is 2n − 1. peg 0 to 1 and another recursive call is
made for ∣ ∣ 1 3 4 6 7 → ∣ 1 3 4 6 7 ∣ , which
Exercise 3.2 Hint: Assume that n ≥ 3 and
is equivalent to a regular P0-type task for
distinguish three cases for the initial state
Ð
→ 5 discs, needing 31 moves.
σ ∈ V (H n
3 ):
Back in the main loop after 44 moves,
• Disc n lies on bottom of tn . disc 10 is transferred from peg 2 to 0 and
• sn ≠ tn and either sn−1 = 3−sn −tn another best buffer disc
or n does not lie on bottom of sn . ′
b = bb (1; ∅, D(2)) = bb (1; ∅, {5}) = 6
• sn ≠ tn , sn−1 ≠ 3 − sn − tn , and n
lies on bottom of sn .
is chosen, leading to a recursive call for
Make use of Lemma 3.2 and Theorem 3.5. ∣ 1 3 4 ∣ 5 → ∣ 1 3 4 5 ∣ , a regular task
4
Exercise 3.3 The optimal solutions are, re- equivalent to 2111 → 1 , taking 15 moves.
spectively, Move 61 is now made by disc 10 from 0
to 2. Finally, the algorithm is called again
4 7
53 ∣ ∣ 421 Ð
→ 3 ∣ 124 ∣ 5 = 21011 Ð → 214 , for ∣ 1 3 4 5 6 7 8 9 2 ∣ → 29 , a stan-
1 6 4 15 4 dard case P3 task which is solved similarly
53 ∣ ∣ 421 Ð
→ 3 ∣ 5 ∣ 421 Ð
→ 1234 ∣ ∣ 5 = 20 Ð→ 21 , by the algorithm taking another 382 moves,
53 ∣ ∣ 421 Ð
1 2
→ 3 ∣ 5 ∣ 421 Ð
6
→ 5 ∣ ∣ 3421 Ð
→ 21 .
4 the relatively large number stemming from
the fact that intermediate states are much
The figures on top of the arrows indi- more regular by now. Altogether 443 moves
cate the move numbers of the intermediate have been made. (A path using peg 1 in-
steps. So the lengths of the respective paths stead of the special peg for the intermediate
are 11, 22, and 9. It turns out that the path move of disc 10 takes at least 710 moves.)
where disc n moves from peg 0 to 1, back to
0, and finally to its goal peg 2 represents the Exercise 3.5 The upper bound follows im-
unique optimal solution. This shows that mediately from Theorem 3.5. For the lower
they do come back (cf. Lemma 2.26)! bound, note that for 1/12 of all states we
have sn−1 = j = sn with disc n above disc
Exercise 3.4 We are asked to get from σ = n − 1. In these (special) cases, a perfect
6 3 8 ∣ 4 1 7 9 2 ∣ 10 5 ∈ T10 to 210 ∈ T 10 (n − 1)-tower must be moved after the sec-
Ð→
in H 103 , which is a special case task. There- ond move of disc n, which takes precisely
fore, in the first call of Algorithm 13, n′ is 2n−1 − 1 moves of discs from [n − 1].
identified to be 9, such that ν = 0 and con- Numerical experiments (cf. [102,
sequently i = sp(σ) = 0, k = 1. The best Tabelle 4]) indicate that dn ≈ a⋅2n for large
buffer disc b is n with an a between 0.3 and 0.4.
bb (1; D(0), 10↑) = bb (1; {3, 6, 8}, ∅) = 9
such that a recursive call of the algo- Chapter 4
rithm is made for the standard case task
6 3 8 ∣ 4 1 7 ∣ → ∣ 1 3 4 6 7 8 ∣ , where j = 1,
Exercise 4.1 The vertices in question are
i = 0, and k = 2. Here the best buffer is
shown in Figure A.8. With the aid of the
bb (2; 8↑, D(1)) = bb (2; {3, 6}, {1, 4, 7}) , H3 -to-S3 automaton one finds that the
278 Appendix A. Hints and Solutions to Exercises

start vertex corresponds to 0110 in Sier- 2211, respectively. For (d) it suffices to en-
piński labelling. Similarly, the goal vertices ter the first two tits, leading to 20 ∗ ∗ ; for
in (a) to (c) transform into 2100, 2212, and (e), the first three tits become 211∗ .

0000

0002 0001

0012 0021

0011 0022
0010 0020
0211 0122

0210 0212 0121 0120

0220 0202 0101 0110

0222 0111
0221 0201 0200 0100 0102 0112
1222 2111

1221 1220 2110 2112

1201 1210 2120 2102

1200 1211 2122 2100


1202 1212 2121 2101
1100 1011 2022 2200

1102 1101 1010 1012 2021 2020 2202 2201

1112 1121 1020 1002 2001 2010 2212 2221

1111 1110 1120 1122 1022 1021 1001 1000 2000 2002 2012 2011 2211 2210 2220 2222

Figure A.8: Graph H34 for the tasks in Exercise 4.1 with the starting vertex in
green, the goal vertices in red

The first pair of input (0, 2) defines the of the corresponding graph. In Figure A.8
P2-decision automaton, namely with i = 0, the set of those goal states which need two
j = 2, and k = 1. Entering the other pairs moves of disc 4 to be reached from the green
leads to the states with the corresponding vertex 0220 is highlighted in the lower right
capital letter, such that for (a) and (d), disc subgraph; on the green line next to that
4 moves once only, for (c) and (e) it has to area one can find the only two goal vertices
move twice and for (b) there are two opti- with two shortest paths from 0220.
mal solutions including one or two moves
of the largest disc, respectively. Exercise 4.2 If tn+1 = i, then disc n + 1 will
For tasks involving a relatively small not move at all. If tn+1 = k, then we have
number of discs, one might, of course, find to use Romik’s P2 decision automaton in
the type of solution easily by an inspection Figure 2.27 with letters j and k switched
Appendix A. Hints and Solutions to Exercises 279

and we will always end up in state D. If that for any i, j and k, j ≠ k, there is a
tn+1 = j, then we employ the automaton as path between ij n−1 and ikn−1 which goes
it is and realize that, although we might through all vertices from iSpn . Obviously
reach state B eventually if the pair (k, k) that reduces the inductive argument to the
is to be entered, the j in the first position statement that j n and kn , j ≠ k, may be
of all subsequent input pairs will never lead connected in Spn by a path going through all
to either C or E. vertices (for all n). Without loss of general-
By isomorphy, non-perfect initial ity assume j = 0 and k = p−1. By the induc-
states exist for the TH as well such that tive hypothesis we may find a path from 0n
all tasks can be solved optimally with at to 01n−1 through all vertices of 0Spn . Add
most one move of the largest disc. the edge between 01n−1 and 10n−1 to the
path. By the same argument we may find a
Exercise 4.3 Hint: Induction on n.
path from 10n−1 to 12n−1 through all ver-
Exercise 4.4 For n = 1 the proposition is tices of 1Spn . Continue this procedure until
trivial since Sp1 is a complete graph. Let (p−2)(p−1)n−1 is joined to (p−1)(p−2)n−1
n ≥ 2 and consider the sequence of paths and a path from (p −1)(p −2)n−1 to (p −1)n
P0 , P1 , . . . , Pp−1 , where P0 is a path be- through all vertices of (p − 1)Spn is added at
tween the vertices 0(p − 1)n−1 and 01n−1 , the end.
Pp−1 between the vertices (p − 1)(p − 2)n−1
Exercise 4.5 Hint: Determine the size of a
and (p − 1)0n−1 , and for i = 1, 2, . . . , p − 2,
corresponding perfect code using the fact
Pi is a path between the vertices i(i − 1)n−1
that every vertex of it, except possibly some
and i(i + 1)n−1 . We claim that the paths Pi
extreme vertices, cover four vertices.
can be constructed in such a way that they
include all the vertices from iSpn , i ∈ [p]0 . Exercise 4.6 A planar drawing of S42 is given
To prove the claim it is enough to see in Figure A.9.

12 13
11

10

01

00
21 20 02 03 30 31
22 33

23 32

Figure A.9: Planar drawing of S42

Every subgraph ijS41 of S43 with i, j ∈ tices induce a K5 -subdivision in S43 . Hence
[4]0 , i ≠ j, is isomorphic to K4 . Adding all S4n with n ≥ 3 are non-planar and so are
a vertex ikj with i ≠ k ≠ j it can easily Spn with p > 4 and n ≥ 1 since they contain
be seen from Figure 4.5 that these five ver- a K5 .
280 Appendix A. Hints and Solutions to Exercises

Exercise 4.7 Hint: Mimic the proof of pegs, 1 + x move at least twice, such that
Proposition 2.22. 2(1 + x) + 2 = 2(2 + x) moves are necessary.
Exercise 4.8 First check the cases p = 1, 2. Exercise 5.3 First select k non-empty pegs
For p ≥ 3, the p-cliques sSp1 (cf. Theo- in (kp) ways. Top discs for these pegs can
rem 4.3) cover all vertices of Spn . As there be selected in (n−1
k−1
) ways because disc 1 is
are pn−1 such cliques and each can con- always one of them. After the top discs are
tain at most one vertex of an independent selected, there are k! ways to assign them
set, α (Spn ) ≤ pn−1 . On the other hand, to the non-empty pegs.
it is well-known (and not difficult to see)
Exercise 5.4 This is proved by induction
that χ(G) ≥ ∣G∣/α(G); cf. [336, Proposi-
on ν. Quite surprisingly, the case ν = 0
tion 5.1.7]. Since χ(Spn ) = p it follows that
is not trivial as usual but requires another
α (Spn ) ≥ pn−1 . (A direct argument for the
straightforward induction on q to get
latter inequality would be to observe that
q
vertices of the form sn sn−1 . . . s2 0 are not i q q
∑(−2) ( ) = (−1) .
adjacent; cf. the canonical vertex-coloring.) i=0 i
The induction step is then by showing that
Chapter 5 both polynomials fulfill the recurrence re-
lation
Exercise 5.1 Let x = 0 (red case in Fig- 1 ν+q
ure 5.3). Then mn = ∆ν−1 . For m = mn , P (ν + 1) = (P (ν) + ( )) ,
2 q
we have µ + (y ≠ 0) = ν − 1 < ν = n − m. If
m < mn , then µ + (y ≠ 0) < ν − 1 + 1 = ν < which in turn follows from the standard
n−m. For m > mn , necessarily µ = ν −1 and properties of combinatorial numbers.
y ∈ [ν − 1], such that µ + (y ≠ 0) = ν − 1 + 1 = Exercise 5.5 Let s be an arbitrary regu-
ν > ν − y = n − m. lar state and consider a sequence M of
Turning to the case x ≠ 0 (green in moves in which each disc is moved at least
Figure 5.3), mn has to be written as ∆ν , once. Just before disc n is moved, discs
if x = ν, and ∆ν−1 + x otherwise. In the 1, 2, . . . , n − 1 are on the same peg. With-
former case we have for m = mn : µ + (y ≠ out loss of generality assume that disc n − 1
0) = ν = n − m, in the latter µ + (y ≠ 0) = is moved after the move of disc n. It fol-
ν − 1 + 1 = ν = n − m. If m < mn , then lows that after the move of n, we need to
µ + (y ≠ 0) ≤ µ + 1 ≤ ν < n − m. So let finally solve a P0 problem for n − 2 discs using
m > mn . Then either µ = ν − 1 and y > x at least 2n−2 − 1 moves and to move disc
or µ = ν and y ∈ [x]0 . In the former case, n − 1. Together with the move of disc n, the
µ + (y ≠ 0) = ν − 1 + 1 = ν > n − m and in the sequence M is of length at least 2n−2 + 1.
latter case µ + (y ≠ 0) = ν + (y ≠ 0) ≥ ν > Hence g(3, n) ≥ 2n−2 + 1. To see that equal-
n − m. ity holds, consider the regular state 001n−2 .
Exercise 5.2 The case ν = 0 is trivial; for Exercise 5.6 Let k ∈ [p] and consider the
ν = 1 it is clear that each disc moves at regular states in which exactly k pegs are
least once. For ν = 2 we have n = 3 + x non-empty. We can select k pegs in (kp)
discs, x ∈ [3]0 , which can be transferred to ways. The n discs can be partitioned into k
two pegs in 2(2+x) moves. But this number non-empty parts in { nk } ways, these parts
is also needed, because if all discs go to one can be distributed onto k pegs in k! ways.
and the same peg, then 2 + x of them have Hence there are exactly (kp) { nk } k! = { nk } pk
to move at least twice, resulting in at least regular states with exactly k non-empty
2(2 + x) + 1 moves. With 3 + x discs on two pegs.
Appendix A. Hints and Solutions to Exercises 281

Consider next an arbitrary regular Now start in the state 01n of Hp1+n and
state with k non-empty pegs. There are transfer the tower of n smallest discs on
k(p − k) legal moves between non-empty hamiltonian paths according to the above
and empty pegs. Moreover, between two in a cyclic fashion from peg to peg, each
non-empty pegs, exactly one move is legal. complete transfer being followed by a sin-
We conclude that the degree of an arbi- gle move of disc n + 1 to the next peg in the
trary regular state with k non-empty pegs same direction.
is k(p − k) + (k2) = 12 k(2p − k − 1), the extra
factor 12 in the desired formula stemming Exercise 5.8 Assume that one of the p(p−1)
from double-counting of edges. non-perfect states, say 01, is a codeword.
Then 2Hp1 contains no codeword, because
Exercise 5.7 Hint: First use induction on
otherwise 21 would be covered twice. But
n to show that there is a hamiltonian
this means that 22 is not covered.
path starting and ending in distinct per-
fect states. (Say, move disc n + 1 stepwise Being surrounded by non-codewords,
from peg 0 to 1, from peg 1 to peg 2, and all perfect states must be codewords them-
so on to p − 1. Before each such move, a selves. This results in a (unique) perfect
hamiltonian path in the corresponding n code.
discs subgraph can be performed to trans- Exercise 5.9 Hint: Look at the 24 flat states,
fer the tower consisting of the n smallest i.e. those where the three discs are dis-
discs to a peg allowing disc n + 1 to move. tributed on three different pegs.
Note that this sequence visits all the states The subgraph of H43 shown in Fig-
in which disc n + 1 is on a fixed peg. Act ure A.10 is a subdivision of K3,3 . Therefore,
similarly after the last move of disc n + 1.) by Kuratowski’s theorem, H43 is not planar.

210 102 021

302 132
213 023
230 310 103 031 321

312 032 203 320 130


013 123
231 301

012 201 120

Figure A.10: K3,3 -subdivision in H43

Exercise 5.10 The proof of Proposition 5.40 position. (If there is no such disc, then t is
is by induction on n. The cases n = 0 reached and each disc moved at most once.)
and n = 1 are clear. Now let s, t ∈ [p]n+1
0 , Move it to peg k. Now peg d − 1, the goal of
n ∈ [p − 2], with ∀ d ∈ [n + 1] ∶ td = d − 1. disc d, is not empty and all discs on it are
Starting in s, move as many discs as pos- not in their end position. We may therefore
sible directly to their goal pegs. Then, by consider the task which remains after ignor-
virtue of Lemma 5.39, some peg k ≥ n + 1 ing pegs k and d−1 and all discs lying there.
ν
is empty in the resulting state. Let d be This is a task in Hp−2 with ν ∈ [n]0 of the
any top disc which is not already in its end same type as the original one. (The cases
282 Appendix A. Hints and Solutions to Exercises

p = 3 and 4 can easily be sorted out.) By in- Now ε(s) ≥ n + ⌊ n2 ⌋ for every s, such
duction assumption, this can be solved with that rad(Hpn ) ≥ n + ⌊ n2 ⌋.
at most ⌊ ν2 ⌋ ≤ ⌊ n−1
2
⌋ discs moving precisely For (b) we deduce from Proposi-
twice and all other discs moving at most tion 5.40 that
once, those in end position being fixed. All
n n n
discs on peg d − 1 can now be moved di- ∃ t ∈ [p]0 ∀ s ∈ [p]0 ∶ d(s, t) ≤ n + ⌊ ⌋ ,
rectly to their goals and finally disc d too 2
in its second move. The former ones there- such that there is a t with ε(t) ≤ n + ⌊ n2 ⌋,
fore all moved precisely once, the latter one
whence rad(Hpn ) ≤ n + ⌊ n2 ⌋.
twice; no disc in end position was moved.
Note that t and in fact all flat states
Since ⌊ n−1 ⌋ + 1 = ⌊ n+1 ⌋, Proposition 5.40 is
2 2 belong to the center of Hpn if n < p. The
proved.
sequence xn = n + ⌊ n2 ⌋ is [296, A032766]; it
For (a) we show that
fulfills the recurrence
n n n
∀ s ∈ [p]0 ∃ t ∈ [p]0 ∶ d(s, t) ≥ n + ⌊ ⌋ . x0 = 0, x1 = 1, ∀ n ∈ N ∶ xn+2 = xn + 3 .
2

For s ∈ [p]n
0 let ∀ d ∈ [⌈ 2 ⌉]:
n
Exercise 5.11 Hint: Observe that the states
in question are precisely the states in which
td = (sd + (sd = sn−d+1 )) mod p = tn−d+1 . disc n is alone on peg i and j is the only
empty peg. (For the sequence, cf. [296,
Then all discs from [n]∖[⌊ n2 ⌋] move at least
A180119].)
once, the others at least twice, summing up
to at least n + ⌊ n2 ⌋ moves. Exercise 5.12 See Figure A.11.

Figure A.11: Optimal solutions for 0233 → 3001

Chapter 6 two smallest discs is at least one more than


the number of moves of all the other discs.
Exercise 6.1 Hint: Observe that in any op- Then use induction to reach the required
timal sequence of moves, after a move of conclusion.
a disc that is not one of the two smallest,
the next move must be either a move of Exercise 6.2 A solution making 11 moves
the smallest white disc or a move of the is presented below. By (6.2), the solution
smallest black disc. Also, any solution must is optimal. To distinguish the colors of the
start and end with a move of a smallest three stacks we denote their discs by d, d′ ,
disc. Hence the number of moves of the and d′′ , d ∈ {1, 2}, respectively.
Appendix A. Hints and Solutions to Exercises 283

1 1′ 1′′
2 2′ 2′′
→ and Romik’s automaton in Figure 2.27 tells
1 1′′ us that the largest disc 8, i.e. the largest
2 2′ 1′ 2′′
→ golden disc, moves twice in the shortest
1
→ path, i.e. we have a class II task. Before the
2 2′ 1′ 2′′ 1′′
first move of disc 8, disc 7 is not moved, be-
1
2 2′ 1′ 2′′ 1′′
→ cause it is not in the way. Between the two
1 moves of disc 8, the 7-tower of smaller discs
2 2′ 1′ 2′′ 1′′
→ is transferred with one move of disc 7. This
1
→ is followed by the last move of disc 8. After
2′′ 2 2′ 1′ 1′′
that the 7-tower is decomposed into its final
1
2′′ 2 2′ 1′ 1′′
→ distribution with no move of disc 7 because
1
it is already in its terminal position and
2′′ 2 1′ 2′ 1′′
→ the starting configuration is perfect. Thus,
1
→ the largest silver disc moves only once, al-
2′′ 2 1′ 2′ 1′′
though it is the “lighter” one. The shortest
1
2′′ 2 1′ 2′ 1′′
→ path needs 208 moves.
1 1′ Exercise 6.6 With the aid of the H3 -to-S3
2′′ 2 2′ 1′′

′′ ′ automaton of Figure 4.4 the task translates
1 1 1
2′′ 2 2′ into the task s ∶= 02020202 → 12121212 =∶ t
in S38 . Romik’s automaton in Figure 2.27,
with i = 0, j = 1, and k = 2 moves to state B
Exercise 6.3 By Theorem 6.2 it suffices to on the input (k, k), remains there after the
find two different solutions with 22 moves. next input (i, j) and then, on (k, k) again,
changes to state E, such that disc 8 neces-
Exercise 6.4 As in the case of an even num- sarily moves twice. It follows that
ber of discs, the optimal solution for the
task 1(01)n → 0(10)n can be decomposed d(s, t) = d(s, 027 ) + 1 + (27 − 1)
into the transfer of 2n smaller discs to peg
+ 1 + d(127 , t) = 213 .
2, the single move of the extra disc to peg
0 and the sorting of the 2n-tower on 2 into The first sub-task is 201201 → 16 in H36 .
odd and even on pegs 0 and 1, respectively. The P1-automaton in Figure 2.6 yields 1 as
This time, the first three input pairs are the idle peg of the best first move, such that
(1, 0), (0, 1), and (1, 0) for the Hanoi la- disc 2 moves from peg 0 to peg 2. (With
belling, leading to (1, 0), (2, 2), (0, 1) in the only one move of the largest disc, the full
Sierpiński notation, such that in the P2 de- 255 moves are needed; in that case, disc 1
cision automaton i = 1, j = 0, and k = 2 moves to peg 2 first, such that this would
and again the two input pairs are (k, k) destroy optimality already.)
and (j, i). Therefore, the total number of
moves is 2y2n +1 = ⌊ 57 22n+1 ⌋+(n mod 3 = 2). Exercise 6.7 Since t = 2 = r and N = 3, The-
Putting the results together, the swapping orem 6.3 asserts that the optimal number
of the positions of N ∈ N discs, initially of moves is 29, which is indeed the length
sorted according to parity, needs 2yN−1 +1 = of the given path. The sequence of moves
⌊ 75 2N ⌋ + (N mod 3 = 2) moves. differs from the one produced by Algo-
rithm 20 starting from the state 217 ∣ 465 ∣ 3.
Exercise 6.5 Translated into the language After this state disc 2 is moved while Algo-
of the TH, the task is (01)4 → (1001)2 . rithm 20 moves disc 3.
Applying the H3 -to-S3 automaton of Fig-
ure 4.4, this becomes 0(210)2 2 → (1221)2 ,
284 Appendix A. Hints and Solutions to Exercises

Chapter 7 Exercise 7.2 a) The correspondence be-


l
Exercise 7.1 As G is hamiltonian, G−X has tween s and σ is given by sn+l = l + ∑ σk
at most ∣X∣ connected components consist- k=1

ing of the sections of an arbitrary hamilto- for all l ∈ [p + 1]0 . Figure A.12 refers to this
nian cycle C between the removed vertices situation: on the left one can see the posi-
of X. Hence if two vertices of X would be tions sn+k of peg bottoms dividing the set
consecutive on C, G − X would consist of of n (uncolored) balls, and on the right the
at most ∣X∣ − 1 connected components. corresponding distribution of the balls on
For G = L, consider X as the set of the pegs is depicted.
the 12 vertices of degree 4.

1 k p
h1
{ hk
{ { hp
{
{

{
0=sn+0 sn+1 sn+k-1 sn+k sn+p-1 sn+p=n+p 1 k p

Figure A.12: Correspondence between positions of peg bottoms and the distribu-
tion of balls

b) We have, in fact for n ∈ Z, cases; cf. Exercise 7.3. As an example we


p
take the classical London graph L, where
n p
lh = ∣{σ ∈ N0 ∣ σ ≤ h, ∑ σk = n}∣ , p = 3 = n and h = 123, such that
k=1
3 3 ν µ
3 ν
where σ ≤ h is meant componentwise, l123 = ∑ l12 =∑ ∑ ∑ (λ = 0)
ν=0 ν=0 µ=ν−2 λ=µ−1
i.e. σk ≤ hk for every k ∈ [p]. (Of course,
3 ν
lhn = 0 for n < 0.) Then for p = 0 we obvi-
= ∑ ∑ (µ ≤ 1) = 1 + 2 + 2 + 1 = 6.
ously get ν=0 µ=ν−2
n
l∅ = (n = 0). (A.3)
c) Each σ corresponds to a distribution of
Moreover, if g ∈ Np0 and α ∈ N0 , then
n uncolored balls on the pegs. Then there
n
n are n! different colorings of the balls.
lgα = ∑ lgν ; (A.4)
ν=n−α Exercise 7.3 a) Induction on p. For p = 1,
p we have
this is because necessarily σp+1 = n − ∑ σk n
1+n−1
p
k=1 lnn = ∑ l∅ν
=1=( ).
ν=0 n
and therefore ∑ σk runs through all (non-
k=1 Furthermore,
negative) values from n − α to n.
With equation (A.4), we can break n n
p+ν −1
down lhn recursively to a sum consisting lnnp+1 = ∑ lnnp = ∑ ( )
µ ν=0 ν=0 ν
only of terms of the form l∅ , which can then
be evaluated using equation (A.3). Explicit p+1+n−1
=( ),
formulas for lhn can only be given in specific n
Appendix A. Hints and Solutions to Exercises 285

the latter by induction on n. It follows that where in the fourth step use has been made
p+n−1 (p + n − 1)! of formula (0.6).
∣Opn ∣ = n!( )= .
n (p − 1)!
Exercise 7.5 a) The steps from the proof of
b) Of all permutations s ∈ Symn+p only Theorem 7.3 are illustrated in Figure A.13.
those with sp+n = p + n are admissible, b) Since no ball is in right position at the
i.e. at most ∣Symn+p−1 ∣ = (p + n − 1)!. Only start, it is clear that each one has to move
1/(p − 1)! of these fulfil the necessary con- at least once and ball 3 at least twice. More-
dition of strict monotonicity sn+1 < sn+2 < over, an easy combinatorial analysis shows
⋯ < sn+p−1 < sn+p . But the latter also im- that if ball 2 moves once only, then 4 has to
plies k ≤ sn+k ≤ n + k for all k ∈ [p + 1]0 , move three times and if ball 4 moves only
whence 1 ≤ sn+k − sn+k−1 ≤ n + 1 for all once, then 2 moves thrice. Therefore, an op-
k ∈ [p]. Therefore the condition in Defi- timal solution needs at least 7 steps. Here
nition 7.2 is automatically fulfilled for all is one:
remaining permutations.
1∣2∣43 → 21∣ ∣43 → 21∣4∣3 → 1∣24∣3 →
Exercise 7.4 Choose q ∈ [p] pegs. The
number of states with all balls distributed 1∣324∣ → ∣324∣1 → ∣24∣31 → ∣4∣231 .
among all these pegs is, for n ≥ q,
Exercise 7.6 The lower bound follows from
n! n−1 1 = g (L3222 ) ≤ cr (L3222 ) . For the upper
∣Oqn−q ∣ = n!( ).
(n − q)! q−1 bound, see Figure A.14. Better bounds are
n!
This is so because there are (n−q)! choices not known so far.
to fill the bottom line of balls on all q pegs,
and the remaining n − q balls can be dis- Exercise 7.7 This can be done by adding all
tributed arbitrarily, such that the number missing vertices and their adjacent edges to
of these distributions can be taken from the the drawing of L3222 in Figure 7.9 without
previous exercise. introducing any crossings. In Figure A.15
Each of these states has degree q(p − the mixed graph of Lucas’s second problem
Ð→
1), whence by the Handshaking lemma for 3 discs H 33 is drawn on the torus; the
p
underlying graph, completed by the edges
n 1 p n−1 in red, is O33 . In fact, we know from Exer-
∥Op ∥ = (p − 1) n! ∑ q( )( )
2 q=1 q q−1 cise 7.4 that ∥O3n ∥ = 23 n(n + 1)! = 3∆n n!
p
p−1 n−1 (cf. [102, Satz 3.7]). Finsterwalder noted
= ∆p−1 n! ∑ ( )( ) in [102, Satz 3.8] that 3∆n−1 n! out of the
q=1 q −1 q−1
2∥O3n ∥ arcs of the directed Oxford graph
n−1
p−1 n−1 Ð

= ∆p−1 n! ∑ ( )( ) are missing in H n 3 because these moves do
λ=0 λ n −1−λ not obey the divine rule; hence it has only
3
p−2+n 2
n(n + 3)n! arcs. Finally, ∆n−2 n! edges of
= ∆p−1 n!( )
n−1 On3 , n ∈ N, i.e. 6 for n = 3, are missing in the
Ð

p−2+n underlying graph of H n 3 , because the cor-
( = ∆p−1 n!( )) responding moves do not conform with the
p−1
divine rule in both directions; hence, this
n p (p − 2 + n)! graph has (n2 + 3n − 1)n! edges (cf. [102,
= ,
2 (p − 2)! Satz 3.9]).
286 Appendix A. Hints and Solutions to Exercises

(0)
! 2
5 4 3
1 2 3 5 4 1
2 moves 3 moves
( 1)
3 3
4 5 1 2
1 2 5 4

1 4 2 4
(2)
3 4 5 3
1 5 (3) 1 4
4 2 5 2

1 5 (4.2) 4 1
(3)
3 5 4 3
5 1 ( 4.1) 4 1
4 2 5 2
2 moves
( 4.1)
5 5 2 moves 4
5 1 1 1 4 1
4 2 3 4 2 3 2 3 5 2 3

(4.2)
3 2 moves 2 moves 3
4 1 4 1 4 1 1 4
5 2 5 2 3 5 2 3 5 2

Figure A.13: Solution for 1 ∣ 2 ∣ 43 → ∣ 4∣ 231 in L4234

Exercise 7.8 Let us first determine the fixed a fixed point of (χ, π) if both χ and π are
points for the action of (Γ33 , ⋅ , 133 ) on rotations (derangements) or both are reflec-
V (L3222 ). Apart from the identity 133 , only tions. The distribution of fixed points to the
“flat” distributions of the three balls can group elements is shown in Table A.1.
lead to a fixed point, and they can only be
Appendix A. Hints and Solutions to Exercises 287

3||21

23|1| 23||1
|2|31
3|1|2
3|2|1
3|21|
|1|32 |32|1
13||2 13|2|
1||32 3||12 3|12| 1|32|
|31|2

|3|12 |12|3
31||2 31|2|
1|3|2 1|2|3
21|3| 21||3
|13|2 |2|13

|21|3
2|13| 2||13
1|23| 1||23
12|3| 12||3
|23|1 |1|23
2|31|
2|3|1
2|1|3
32||1 32|1|
|3|21
2||31

Figure A.14: The graph L3222 with 8 crossings in the plane

We can read the following from that and the other one of the 6 flat distributions
table. The sum of the sizes of the fixed of balls, each of which is fixed under the ac-
point sets is tion of 6 group elements. We therefore only
3 g have to consider two representatives, 12 ∣ 3 ∣
∑ ∣V (L222 ) ∣ = 42 + 4 × 3 + 9 × 2 = 72 , and 1 ∣ 2 ∣ 3, say. They have degrees 3 and 6,
g∈Γ33
respectively, such that the statements re-
such that Burnside’s lemma Corollary 0.9 lating to degrees are obvious.
yields (with ∣Γ33 ∣ = 3! × 3! = 36) that the
number of equisets of vertices is just 2. We now turn to the conclusions about
From Theorem 0.8 we learn that one of the metric properties of L3222 . The only tedious
classes consists of the 36 non-flat states, step is to determine all distances to other
which stay fixed only under the identity, vertices from the two prototype states,
288 Appendix A. Hints and Solutions to Exercises

123||
|321|

3|21| 3||21
|31|2 23|1| 23||1 |2|31 ||231
||132 |132|
3|1|2 3|2|1
|1|32 |32|1
231|| ||312 13||2 13|2| |312|
1||32 1|32|
3||12 3|12|

|3|12 |12|3
31||2 213|| 31|2|
21|3| |213| ||213 21||3

1|3|2 |13|2 |2|13 1|2|3

1|23| 2|13| 2||13 1||23


321||
|23|1 2|3|1 12|3| 12||3 2|1|3 |1|23

|123| ||123
||321

|3|21 312|| |21|3


2||31 32||1 32|1| 2|31|

132|| |231|

Ð

Figure A.15: The mixed graph H 33 and extra edges of O33 on the torus

i.e. to span shortest-path trees rooted in attained, e.g. for the task 1 ∣ 2 ∣ 3 → 3 ∣ ∣ 12.
12 ∣ 3 ∣ and 1 ∣ 2 ∣ 3. This can be done, e.g., (From the Handshaking lemma it also fol-
by recourse to Figure A.14. lows that ∥L3222 ∥ = 72.) Moreover, the sums
In Figure A.16 all distances from the of lengths of all shortest paths in the trees
respective start vertices are indicated. It are 140 and 133, respectively. This leads to
turns out that the height of both trees is the total sum of 36 ⋅ 140 + 6 ⋅ 133 = 5838 and
5, which proves the statements about ec- the average distance of 13941
.
centricities and diameter, the latter being
Appendix A. Hints and Solutions to Exercises 289

π
123 231 312 132 321 213
χ

123 V

231

312

132

321

213

Table A.1: Fixed point sets for symmetries on V (L3222 )

Finally, in order to arrange tasks into puts them into equisets of size 36.
equisets, we have to let the group act on
the set V (L3222 )×V ˙ (L3222 ). A task (s, t) can
only be invariant under the action of a
group element g, if both s and t are in the Chapter 8
fixed point set of g. This means that there
are just 2 fixed points for each combina- Exercise 8.1 The number of moves of disc
tion of reflections and 6 for the combina- d is 2 ⋅ 3n−d .
tions of non-trivial rotations. That makes
a total of 1722 + 4 ⋅ 6 + 9 ⋅ 2 = 1764, such that Exercise 8.2 Hint: Every move changes the
Corollary 0.9 tells us that there are 49 eq- parity of the number of discs on peg 1.
uisets of tasks. Moreover, Table A.1 shows
that tasks being fixed by a non-trivial rota- Exercise 8.3 It suffices to prove that the
tion are fixed by another one, whereas those tower of n discs can be moved from peg 0
fixed by a reflection are not invariant under to peg 1. Design an algorithm for this task
any other non-trivial group element. There- and prove its correctness by induction.
fore, the former constitute the only equiset
of size 12 the latter the only one of size Exercise 8.4 First show by induction that
18. Representatives of these two classes are, [n] [n] [n] [n]
∣0 Ð→ 2∣ = ∣2 Ð→ 1∣ and ∣1 Ð→ 2∣ = ∣2 Ð→ 0∣.
e.g., 1 ∣ 2 ∣ 3 → 2 ∣ 3 ∣ 1 and 1 ∣2 ∣ 3 → 2 ∣ 1 ∣ 3, re- [n] [n]
spectively. All other task are invariant only Setting an = ∣0 Ð→ 1∣, bn = ∣0 Ð→ 2∣,
[n] [n]
under the identity, such that Theorem 0.8 cn = ∣1 Ð→ 0∣, and dn = ∣1 Ð→ 2∣, we read
290 Appendix A. Hints and Solutions to Exercises

3 4
3 3
5 5 4
4
5 5 4
3
4 4 4 4
5 2
5 4 5 3
3 5 5 5
4 4 2 1
4 5
2 4 5 1
4 4
3 3 1
4 1|2|3
3 2 1
4 4 3 1
2 5
3 1 2 3 3 1
3 5 2 4
12|3|
4 1 3
2 2
4 3 4 3
1 6x1= 6
3 6 x 2 = 12
5 2
2 5 3 4 9 x 3 = 27
12 x 4 = 48
2 5 3x1= 3 8 x 5 = 40
2 7 x 2 = 14
4 10 x 3 = 30 41 133
12 x 4 = 48
9 x 5 = 45
41 140

Figure A.16: L3222 with distances from vertex 1 ∣ 2 ∣ 3 (green) and from vertex 12 ∣ 3∣
(blue)

[n] [n] [n]


from Algorithm 23: ∣1 Ð→ 2∣, cn = ∣2 Ð→ 0∣, and ∣1 Ð→ 0∣ = dn =
[n]
an = 2bn−1 + 1, ∣2 Ð→ 1∣, we get:
bn = 2bn−1 + dn−1 + 2, an = 2an−1 + cn−1 + 2,
cn = 2dn−1 + 1, bn = an−1 + dn−1 + 1,
dn = bn−1 + cn−1 + 1 . cn = 2dn−1 + 1,
The initial conditions are left to the reader. dn = bn−1 + cn−1 + 1 .
Exercise 8.5 This is done as in Exercise 8.4. The base of the asymptotic exponential be-
[n] [n]
Now, with an = ∣0 Ð→ 2∣, ∣0 Ð→ 1∣ = bn = havior of these sequences is then the great-
Appendix A. Hints and Solutions to Exercises 291

est (with respect to absolute value) solution an = 3an−1 + 2 for n ≥ 1. This leads to
of λ3 −λ2 −4λ+2 = 0, which is approximately an = 3n − 1. To get a solution using only
2.342923. 18 moves (instead of the 26 needed by the
algorithm) use the fact that when disc 3 is
Exercise 8.6 The first assertion follows from
moved from peg 0 to peg 1, discs 1 and 2
the definitions of legal moves in TH and in
Ð→ need not be both on peg 2; and similarly in
TH( C 3 ), for the second use induction on the later stages.
the number of discs.
Exercise 8.9 Hint: It is easy to see that for
Exercise 8.7 Hint: Consider the states of
any vertex v of a moderately enhanced cy-
the puzzle as the points of the lattice [p]0 ×
cle D, the digraph D − v cannot contain a
[p]0 . Optimal solutions of the puzzle corre-
strong subgraph on at least three vertices.
spond to paths in the grid between (0, 0)
For the converse suppose that D has no
and (p − 1, p − 1) that meet a diagonal point
vertex w such that the digraph D − w con-
(i, i) for precisely one i ∈ [p − 2]. Then take
tains a strong subgraph on at least three
a closer look at the shape of these paths.
vertices and consider two cases. The first
(For details, see [174, p. 375f].)
is that there are vertices u, v ∈ V (D) such
Exercise 8.8 Let an be the number of that (u, v) ∈ A(D) and (v, u) ∉ A(D), the
moves. It is easy to see that a0 = 0 and second being that this is not so.
Glossary

adjacency matrix of a graph A matrix given by


V (G) × V (G) → B, (u, v) ↦ ({u, v} ∈ E(G)) .

binary tree A rooted tree where every vertex has at most two adjacent vertices
which are further away from the root.
bit Short for binary digit.
bridge Edge whose deletion increases the number of connected components of
a graph. Also called cut-edge. In particular, edge whose deletion leaves a
connected graph disconnected.
center The set of those vertices of a graph whose eccentricities are equal to its
radius.
(vertex) chromatic number The minimum number of colors needed for a coloring
of vertices of a graph such that adjacent vertices receive different colors.
circuit Another word for a closed trail.
clique in a graph A complete subgraph.
complete graph A graph G whose edge set is E(G) = (V (G)
2
).
connected graph A finite or an infinite graph is connected if any two of its vertices
are joined by a finite path.
connectivity The connectivity of a graph G is the smallest size of a vertex set X
such that G − X is disconnected or has only one vertex.
cycle in a graph A finite path in a graph to which an edge joining its end vertices
is added.
cyclic permutation A permutation which moves the elements of a subset cyclically:
c1 ↦ c2 ↦ ⋯ ↦ ck ↦ c1 and leaves the others fixed; its length is k, and it is
denoted by (c1 c2 . . . ck ).
degree The number of incident edges of a vertex v is its degree deg(v). A loop
adds 2 to the degree of a vertex.

A. M. Hinz et al., The Tower of Hanoi – Myths and Maths,


DOI: 10.1007/978-3-0348-0237-6, Ó Springer Basel 2013
294 Glossary

derangement A fixed-point-free permutation.


diameter of a graph The maximum over all distances in a connected graph.
digraph Ordered pair D = (V (D), A(D)) consisting of the set of vertices V (D)
and the set of arcs A(D) which are ordered pairs of vertices.
eccentricity of a vertex The maximum distance between a given vertex and all
the other vertices.
edge coloring A function from the edge set of a graph to a set of colors such that
any pair of incident edges receives different colors.
eulerian circuit A circuit in a graph G which contains all edges of G.
eulerian graph A graph which contains an eulerian circuit.
eulerian trail A trail in a graph G which contains all edges of G.
forest A graph F without cycles. It has ∣F ∣−∥F ∥ components [327, Theorem I.35].
geodesic A shortest path between two vertices of a graph G is called a geodesic
in G.
graph Ordered pair G = (V (G), E(G)) consisting of the set of vertices V (G) and
the set of edges E(G) which are unordered pairs of vertices.
graph distance The minimum length of the paths between two vertices of a con-
nected graph.
hamiltonian cycle A cycle containing all the vertices of a given graph.
hamiltonian path A path containing all the vertices of a given graph.
Handshaking lemma Asserts that the sum of the degrees in a graph is twice the
number of its edges.
height of a rooted tree The largest distance from the root.
inclusion-exclusion principle For two finite sets X and Y , ∣X∪Y ∣ = ∣X∣+∣Y ∣−∣X∩Y ∣.
The principle has an extension to any finite family of finite sets.
independence number The maximum size of a subset of vertices of a graph which
are pairwise non-adjacent.
Kuratowski’s theorem Asserts that a graph is planar if and only if it does not
contain a subgraph that is a subdivision of K3,3 or K5 .
length of a path The number of its edges.
loop An edge in a multigraph whose end-vertices are identical, i.e. a singleton.
matching A set of pairwise non-adjacent edges of a graph.
Glossary 295

mixed graph Triple G = (V (G), E(G), A(G)) consisting of the set of vertices
V (G), the set of edges E(G) which are unordered pairs of vertices, and
the set of arcs A(G) which are ordered pairs of vertices.
multigraph Ordered pair G = (V (G), F (G)) consisting of the set of vertices V (G)
and the family F (G) of edges which are singletons or unordered pairs of
vertices.
natural number A non-negative integer. The set of natural numbers is denoted by
N0 , while N stands for the set of positive integers.
order of a graph The number ∣V (G)∣ of the vertices of a graph G; abbreviated
∣G∣.
partition of a set P ⊂ 2V is a partition of the set V , if ∅ ∉ P , V = ⋃ P , and
∀ {X, Y } ∈ (P2 ) ∶ X ∩ Y = ∅.
path graph A graph with one vertex and no edge or a connected graph with two
vertices of degree 1 and the other vertices of degree 2.
path in a digraph A subdigraph with vertex set {xi ∣ i ∈ [` + 1]0 } of size ` + 1 and
arc set {(xi−1 , xi ) ∣ i ∈ [`]}, where ` ∈ N0 is its length, x0 its start vertex and
x` its end vertex.
path in a (multi)graph A subgraph isomorphic to a path graph.
pendant vertex A vertex of a graph of degree 1.
perfect matching A matching M of a graph G such that every vertex of G is
saturated, i.e. is incident with (precisely) one element of M .
periphery The set of those vertices of a graph whose eccentricities are equal to its
diameter.
permutation A bijective mapping from a finite set to itself.
pigeonhole principle There is no injective mapping from [n] to [m] if N0 ∋ m <
n ∈ N (as can be proved by induction on m).
pit Short for p-ary digit, the entities of the number system with base p; cf. bit
and tit.
radius of a graph The minimum over all eccentricities of vertices in a connected
graph.
regular graph Graph all of whose vertices have the same degree. Called k-regular
if their degree is k.
root, rooted tree A rooted tree is a tree one of whose vertices has been specified
as its root.
semi-eulerian graph A graph which contains an eulerian trail.
296 Glossary

singleton A set with precisely one element.


size of a graph The number ∣E(G)∣ of the edges of a graph G; abbreviated ∥G∥.
1-skeleton of a polyhedron The graph whose vertex set is the set of vertices of the
polyhedron and where two vertices form an edge if the corresponding vertices
on the polyhedron are the endpoints of an edge.
spanning subgraph A subgraph with the same vertex set as its host graph.
Stirling numbers of the second kind The number of partitions of an n-set into k
nonempty parts; denoted by { nk }.
subdivision A subdivision of a graph is a graph obtained from it by successively
replacing an edge {x, y} ∈ E by two edges {x, z} and {z, y} with a new vertex
z.
suffix of a sequence (finite or not) (xk )k≥K is a subsequence (xk )k≥L for some
L ≥ K.
tit Short for ternary digit.
total coloring A vertex coloring together with an edge coloring such that each
edge receives a color different from the colors of its end vertices.
total eccentricity The sum of eccentricities ε(v) over all vertices v of a (connected)
graph.
(closed) trail A (closed) walk containing no repeating edge.
transposition A cyclic permutation of length 2.
tree A connected graph without cycles, i.e. a connected forest.
vertex coloring A function from the vertex set of a graph to a set of colors such
that any pair of adjacent vertices receives different colors.
(closed) walk A walk is a finite sequence of vertices such that neighbors in the
sequence are adjacent in the graph. It is closed if the first and last member of
the sequence coincide. In a multigraph it might be necessary to specify which
edge is joining two vertices. Also the subgraph induced by these vertices and
edges.
Bibliography

[1] Afriat, S. N., The Ring of Linked Rings, Duckworth, London, 1982.
[2] Ahrens, W., Mathematische Spiele, B. G. Teubner, Leipzig, 1907.
[3] Allardice, R. E., Fraser, A. Y., La Tour d’Hanoï, Proceedings of the Edin-
burgh Mathematical Society 2 (1883/4) 50–53.
[4] Allouche, J.-P., Note on the cyclic towers of Hanoi, Theoretical Computer
Science 123 (1994) 3–7.
[5] Allouche, J.-P., Astoorian, D., Randall, J., Shallit, J., Morphisms, Square-
free Strings, and the Tower of Hanoi Puzzle, The American Mathematical
Monthly 101 (1994) 651–658.
[6] Allouche, J.-P., Bacher, R., Toeplitz sequences, paperfolding, Towers of
Hanoi and progression-free sequences of integers, L’Enseignement Mathé-
matique (2) 38 (1992) 315–327.
[7] Allouche, J.-P., Bétréma, J., Shallit, J. O., Sur des points fixes de mor-
phismes d’un monoïde libre, RAIRO Informatique Théorique et Applications
23 (1989) 235–249.
[8] Allouche, J.-P., Dress, F., Tours de Hanoï et automates, RAIRO Informa-
tique Théorique et Applications 24 (1990) 1–15.
[9] Allouche, J.-P., Sapir, A., Restricted towers of Hanoi and morphisms, Lec-
ture Notes in Computer Science 3572 (2005) 1–10.
[10] Allouche, J.-P., Shallit, J., The ubiquitous Prouhet-Thue-Morse sequence,
in: C. Ding, T. Helleseth, H. Niederreiter (Eds.), Sequences and their Appli-
cations, Springer, London, 1999, 1–16.
[11] Allouche, J.-P., Shallit, J., Automatic Sequences, Cambridge University
Press, Cambridge, 2003.
[12] Amarel, S., On representations of problems of reasoning about actions, in:
D. Michie (Ed.), Machine Intelligence 3, Elsevier/North-Holland, Amster-
dam, 1968, 131–171.

A. M. Hinz et al., The Tower of Hanoi – Myths and Maths,


DOI: 10.1007/978-3-0348-0237-6, Ó Springer Basel 2013
298 Bibliography

[13] Anderson, J. R., Albert, M. V., Fincham, J. M., Tracing Problem Solving
in Real Time: fMRI Analysis of the Subject-paced Tower of Hanoi, Journal
of Cognitive Neuroscience 17 (2005) 1261–1274.
[14] Arett, D., Dorée, S., Coloring and Counting on the Tower of Hanoi Graphs,
Mathematics Magazine 83 (2010) 200–209.
[15] Armstrong, M. A., Groups and Symmetry, Springer, New York NY, 1988.
[16] Atkinson, M. D., The cyclic Towers of Hanoi, Information Processing Letters
13 (1981) 118–119.
[17] Aumann, S., Über die Anzahl der Stege auf Geodäten von Hanoi-Graphen,
Diploma thesis, Ludwig-Maximilians-Universität, München, 2009.
[18] Autebert, J.-M., Décaillot, A.-M., Schwer, S. R., Henri-Auguste Delannoy
et la publication des œuvres posthumes d’Édouard Lucas, Gazette des Ma-
thématiciens 95 (2003) 51–62.
[19] Azriel, D., Berend, D., On a question of Leiss regarding the Hanoi Tower
problem, Theoretical Computer Science 369 (2006) 377–383.
[20] Azriel, D., Solomon, N., Solomon, S., On an Infinite Family of Solvable Hanoi
Graphs, ACM Transactions on Algorithms 5 (2009) Art. 13.
[21] Backhouse, R., Fokkinga, M., The associativity of equivalence and the Tow-
ers of Hanoi problem, Information Processing Letters 77 (2001) 71–76.
[22] Bailly, B., Les jeux scientifiques, Cosmos, Revue des sciences, Nouvelle série
(15) 39 (1890) 156–159.
[23] Ball, W. W. R., Mathematical recreations and problems of past and present
times, Second edition, Macmillan, London, 1892.
[24] Ball, W. W. R., Mathematical Recreations & Essays, Macmillan, London,
1956.
[25] Barlow, M. T., Perkins, E. A., Brownian Motion on the Sierpinski Gasket,
Probability Theory and Related Fields 79 (1988) 543–623.
[26] Beck, M., Geoghegan, R., The Art of Proof, Springer, New York NY, 2010.
[27] Bell, E. T., Mathematics Queen and Servant of Science, Mathematical As-
sociation of America, Washington DC, 1987.
[28] Bennish, J., The Tower of Hanoi problem and mathematical thinking, Mis-
souri Journal of Mathematical Sciences 11 (1999) 164–166.
[29] Berend, D., Sapir, A., The Cyclic Multi-Peg Tower of Hanoi, ACM Trans-
actions on Algorithms 2 (2006) 297–317.
[30] Berend, D., Sapir, A., The diameter of Hanoi graphs, Information Processing
Letters 98 (2006) 79–85.
Bibliography 299

[31] Berend, D., Sapir, A., Which Multi-Peg Tower of Hanoi Problems are Ex-
ponential?, Lecture Notes in Computer Science 7551 (2012), to appear.
[32] Berend, D., Sapir, A., Solomon, S., The Tower of Hanoi problem on Pathh
graphs, Discrete Applied Mathematics 160 (2012) 1465–1483.
[33] Berg, W. K., Byrd, D. L., The Tower of London spatial problem-solving
task: Enhancing clinical and research implementation, Journal of Clinical
and Experimental Neuropsychology 24 (2002) 586–604.
[34] Berlekamp, E. R., Conway, J. H., Guy, R. K., Winning Ways for Your Math-
ematical Plays, Vol. 4, Second edition, A K Peters, Wellesley MA, 2004.
[35] Biggs, N., The icosian calculus of today, Proceedings of the Royal Irish
Academy 95A Supplement (1995) 23–34.
[36] Biggs, N. L., Discrete Mathematics, Oxford University Press, Oxford, 2002.
[37] Biggs, N. L., Lloyd, E. K., Wilson, R. J., Graph Theory 1736–1936, Claren-
don Press, Oxford, 1986.
[38] Bode, J.-P., Hinz, A. M., Results and open problems on the Tower of Hanoi,
Congressus Numerantium 139 (1999) 113–122.
[39] Boettcher, S., Gonçalves, B., Anomalous diffusion on the Hanoi networks,
Europhysics Letters 84 (2008) Art. 30002.
[40] Boettcher, S., Gonçalves B., Guclu H., Hierarchical regular small-world
networks, Journal of Physics A: Mathematical and Theoretical 41 (2008)
Art. 252001.
[41] Bondy, J. A., Murty, U. S. R., Graph Theory, Springer, New York NY, 2008.
[42] Bosma, W., Wiskundige Verpoozingen, Nieuw Archief for Wiskunde (5) 10
(2009) 42–47.
[43] Brendel, J., Spanning Trees of Hanoi Graphs, Bachelor thesis, Ludwig-
Maximilians-Universität, München, 2012.
[44] Brocot, A., Calcul des rouages par approximation, Revue Chronométrique
3 (1859-61) 186–194. (Reprinted in: R. Chavigny, Les Brocot, une dynastie
d’horlogers, Antoine Simonin, Neuchâtel, 1991, 187–197.)
[45] Brousseau, B. A., Tower of Hanoi with more pegs, Journal of Recreational
Mathematics 8 (1975/6) 169–176.
[46] Buckley, F., Harary, F., Distance in graphs, Addison-Wesley, Redwood City
CA, 1990.
[47] Calkin, N., Wilf, H. S., Recounting the Rationals, The American Mathemat-
ical Monthly 107 (2000) 360–363.
300 Bibliography

[48] Cantor, G., Ueber eine Eigenschaft des Inbegriffs aller reellen algebraischen
Zahlen, Journal für die reine und angewandte Mathematik 77 (1874) 258–
262.
[49] Cantor, G., Ueber eine elementare Frage der Mannigfaltigkeitslehre, Jahres-
berichte der Deutschen Mathematikervereinigung 1 (1890–91) 75–78.
[50] Cardanus, H., De Subtilitate, Guillaume Rouillé, Lyon, 1551.
[51] Carlitz, L., Single variable Bell polynomials, Collectanea Mathematica 14
(1962) 13–25.
[52] Chan, T.-H., A statistical analysis of the Towers of Hanoi problem, Interna-
tional Journal of Computer Mathematics 28 (1989) 57–65.
[53] Chang, S.-C., Chen, L.-C., Yang, W.-S., Spanning Trees on the Sierpinski
Gasket, Journal of Statistical Physics 126 (2007) 649–667.
[54] Chappelon, J., Matsuura, A., On generalized Frame-Stewart numbers, Dis-
crete Mathematics 312 (2012) 830–836.
[55] Chen, G.-H., Duh, D.-R., Topological Properties, Communication, and Com-
putation on WK-Recursive Networks, Networks 24 (1994) 303–317.
[56] Chen, X., Shen, J., On the Frame-Stewart conjecture about the Towers of
Hanoi, SIAM Journal on Computing 33 (2004) 584–589.
[57] Chen, X., Tian, B., Wang, L., Santa Claus’ Towers of Hanoi, Graphs and
Combinatorics 23 (Supplement 1) (2007) 153–167.
[58] Claus (= É. Lucas), N., La Tour d’Hanoï, Véritable casse-tête annamite,
P. Bousrez, Tours, 1883.
[59] Claus (= É. Lucas), N., La Tour d’Hanoï, Jeu de calcul, Science et Nature
1 (1884) 127–128.
[60] Cook, S., The P Versus NP Problem, in: J. Carlson, A. Jaffe, A. Wiles (Eds.),
The Millennium Prize Problems, Clay Mathematics Institute and American
Mathematical Society, Cambridge MA and Providence RI, 2006, 87–104.
[61] Crowe, D. W., The n-dimensional cube and the tower of Hanoi, The Amer-
ican Mathematical Monthly 63 (1956) 29–30.
[62] Culin, S., Korean games with notes on the corresponding games in China
and Japan, University of Pennsylvania, Philadelphia PA, 1895.
[63] Cull, P., Ecklund, E. F., On the Towers of Hanoi and Generalized Towers of
Hanoi Problems, Congressus Numerantium 35 (1982) 229–238.
[64] Cull, P., Gerety, C., Is Towers of Hanoi Really Hard?, Congressus Numer-
antium 47 (1985) 237–242.
Bibliography 301

[65] Cull, P., Nelson, I., Error-Correcting Codes on the Towers of Hanoi Graphs,
Discrete Mathematics 208/209 (1999) 157–175.
[66] Cyvin, S. J., Gutman, I., Kekulé Structures in Benzenoid Hydrocarbons,
Springer, Berlin, 1988.
[67] D’Angeli, D., Donno, A., Self-similar groups and finite Gelfand pairs, Alge-
bra and Discrete Mathematics 2007 (2) 54–69.
[68] D’Angeli, D., Donno, A., Weighted spanning trees on some self-similar
graphs, The Electronic Journal of Combinatorics 18(1) (2011) Art. 16.
[69] Décaillot, A.-M., L’arithméticien Édouard Lucas (1842–1891): théorie et in-
strumentation, Revue d’Histoire des Mathématiques 4 (1998) 191–236.
[70] Décaillot-Laulanglet, A.-M., Edouard Lucas (1842–1891) : le parcours origi-
nal d’un scientifique français dans la deuxième moitié du XIXe siècle, Thèse
de doctorat, Université René Descartes, Paris, 1999.
[71] Della Vecchia, G., Sanges, C., A Recursively Scalable Network VLSI Imple-
mentation, Future Generation Computer Systems 4 (1988) 235–243.
[72] Diestel, R., Graph Theory, Springer, New York NY, 2010.
[73] Dinitz, Y., Solomon, S., On Optimal Solutions for the Bottleneck Tower of
Hanoi Problem, Lecture Notes in Computer Science 4362 (2007) 248–259.
[74] Dinitz, Y., Solomon, S., Optimality of an algorithm solving the Bottleneck
Tower of Hanoi problem, ACM Transactions on Algorithms 4 (2008) Art. 25.
[75] Domoryad, A. P., Mathematical Games and Pastimes, Pergamon Press, Ox-
ford, 1964.
[76] Dubrovsky, V., Nesting puzzles Part I: Moving oriental towers, Quantum
6(1) (1996) 53–59, 49–51.
[77] Dubrovsky, V., Nesting puzzles Part II: Chinese rings produce a Chinese
monster, Quantum 6(2) (1996) 61–64, 58–59.
[78] Dudeney, H. E., The Canterbury puzzles and other curious problems, E. P.
Dutton, New York NY, 1908.
[79] Dudeney, H. E., Amusements in Mathematics, T. Nelson & Sons, London,
1917.
[80] Dunkel, O., Editorial note, The American Mathematical Monthly 48 (1941)
219.
[81] Edwards, A. W. F., Pascal’s Arithmetical Triangle, Charles Griffin and Ox-
ford University Press, London and New York NY, 1987.
302 Bibliography

[82] Eisenstein, G., Eine neue Gattung zahlentheoretischer Funktionen, welche


von zwei Elementen abhängen und durch gewisse lineare Funktional-
Gleichungen definirt werden, Bericht über die zur Bekanntmachung
geeigneten Verhandlungen der Königlich Preussischen Akademie der Wis-
senschaften zu Berlin, 1850, 36–42.
[83] Emert, J. W., Nelson, R. B., Owens, F. W., Multiple Towers of Hanoi with
a Path Transition Graph, Congressus Numerantium 188 (2007) 59–64.
[84] Emert, J. W., Nelson, R. B., Owens, F. W., Multiple Towers of Hanoi with
a Directed Cycle Transition Graph, Congressus Numerantium 193 (2008)
89–95.
[85] Er, M. C., An iterative solution to the generalized Towers of Hanoi problem,
BIT 23 (1983) 295–302.
[86] Er, M. C., An analysis of the generalized Towers of Hanoi problem, BIT 23
(1983) 429–435.
[87] Er, M. C., The Cyclic Towers of Hanoi: A Representation Approach, The
Computer Journal 27 (1984) 171–175.
[88] Er, M. C., The Generalized Colour Towers of Hanoi: An Iterative Algorithm,
The Computer Journal 27 (1984) 278–282.
[89] Er, M. C., The Complexity of the Generalised Cyclic Towers of Hanoi Prob-
lem, Journal of Algorithms 6 (1985) 351–358.
[90] Er, M. C., The Towers of Hanoi and Binary Numerals, Journal of Information
& Optimization Sciences 6 (1985) 147–152.
[91] Er, M. C., The Cyclic Towers of Hanoi and Pseudoternary Codes, Journal
of Information & Optimization Sciences 7 (1986) 271–277.
[92] Er, M. C., A Time and Space Efficient Algorithm for the Cyclic Towers of
Hanoi Problem, Journal of Information Processing 9 (1986) 163–165.
[93] Er, M. C., A General Algorithm for Finding a Shortest Path between two
n-Configurations, Information Sciences 42 (1987) 137–141.
[94] Er, M. C., A Loopless and Optimal Algorithm for the Cyclic Towers of Hanoi
Problem, Information Sciences 42 (1987) 283–287.
[95] Er, M. C., Counter Examples to Adjudicating a Towers of Hanoi Contest,
International Journal of Computer Mathematics 21 (1987) 123–131.
[96] Erickson, M., Pearls of discrete mathematics, CRC Press, Boca Raton FL,
2010.
[97] Euler, L., Calcul de la probabilité dans le jeu de rencontre, Histoire de
l’Académie Royale des Sciences et Belles Lettres 7 (1753) 255–270.
Bibliography 303

[98] Euler, L., Solutio quaestionis curiosae ex doctrina combinationum, Mémoires


de l’Académie Impériale des Sciences de St. Pétersbourg 3 (1811) 57–64.
[99] ler, L., Pis~ma k Uqenym, V. I. Smirnov (Red.), Izdatel~stvo
Akademii Nauk SSSR, Moskva, 1963.
[100] Faber, A. H., Kühnpast, N., Sürer, F., Hinz, A. M., Danek, A., The iso-
effect: Is there specific learning of Tower of London iso-problems?, Thinking
& Reasoning 15 (2009) 237–249.
[101] Fang, J.-F., Liang, W.-Y., Chen, H.-R., Ng, K.-L., Novel Broadcasting Algo-
rithm of the Complete Recursive Network, Informatica 31 (2007) 131–136.
[102] Finsterwalder, S., Der Turm von Hanoi mit irregulären Anfangszuständen,
Diploma thesis, Ludwig-Maximilians-Universität, München, 2008.
[103] Frame, J. S., Problems and solutions: Advanced problems: Solutions: 3918,
The American Mathematical Monthly 48 (1941) 216–217.
[104] Franci, R., Jealous Husbands Crossing the River: A Problem from Alcuin to
Tartaglia, in: Y. Dold-Samplonius, J. W. Dauben, M. Folkerts, B. van Dalen
(Eds.), From China to Paris: 2000 years transmission of mathematical ideas,
Franz Steiner, Stuttgart, 2002, 289–306.
[105] Fritsch, R., Fritsch, G., The Four-Color Theorem, Springer, New York NY,
1998.
[106] Fu, J.-S., Hamiltonian connectivity of the WK-recursive network with faulty
nodes, Information Sciences 178 (2008) 2573–2584.
[107] Garnier, R., Taylor, J., 100% Mathematical Proof, John Wiley & Sons,
Chichester, 1996.
[108] Gauss, C. F., Mathematisches Tagebuch 1796–1814, 2. Auflage, Akademische
Verlagsgesellschaft Geest & Portig, Leipzig, 1979, 102.
[109] Gedeon, T. D., The Cyclic Towers of Hanoi: An Iterative Solution Produced
by Transformation, The Computer Journal 39 (1996) 353–356.
[110] Gillings, R. J., Mathematics in the Time of the Pharaohs, Dover, New York
NY, 1982.
[111] Glaisher, J. W. L., On the residue of a binomial-theorem coefficient with
respect to a prime modulus, The Quarterly Journal of Pure and Applied
Mathematics 30 (1899) 150–156.
[112] Götz, K. A. M., Äquivalenzklassen von Turm-Aufgaben, Diploma thesis,
Ludwig-Maximilians-Universität, München, 2008.
[113] Graham, R. L., Knuth, D. E., Patashnik, O., Concrete mathematics,
Addison-Wesley, Reading MA, 1989.
304 Bibliography

[114] Graves, R. P., Life of Sir William Rowan Hamilton, Vol. III, Dublin Univer-
sity Press, Dublin, 1889.
[115] Gravier, S., Klavžar, S., Mollard, M., Codes and L(2, 1)-labelings in Sier-
piński graphs, Taiwanese Journal of Mathematics 9 (2005) 671–681.
[116] Gray, F., Pulse code communication, US-Patent Nr. 2,632,058, 1953.
[117] Grigorchuk, R., Šunić, Z., Schreier spectrum of the Hanoi Towers group on
three pegs, in: P. Exner, J. P. Keating, P. Kuchment, T. Sunada, A. Teplyaev
(Eds.), Analysis on Graphs and Its Applications, American Mathematical
Society, Providence RI, 2008, 183–198.
[118] Grigorchuk, R., Šuniḱ, Z., Asymptotic aspects of Schreier graphs and Hanoi
Towers groups, Comptes Rendus de l’Académie des Sciences Paris, Ser. I,
342 (2006) 545–550.
[119] Gros, L., Théorie du Baguenodier, Aimé Vingtrinier, Lyon, 1872.
[120] Guan, D.-J., Generalized Gray Codes with Applications, Proceedings of the
National Science Council, Republic of China - Part A 22 (1998) 841–848.
[121] Guibert, O., Stack words, standard Young tableaux, permutations with for-
bidden subsequences and planar maps, Discrete Mathematics 210 (2000)
71–85.
[122] Guy, R. K., Crossing numbers of graphs, Lecture Notes in Mathematics 303
(1972) 111–124.
[123] de Guzmán, M., The role of games and puzzles in the popularization of
mathematics, L’Enseignement Mathématique (2) 36 (1990) 359–368.
[124] Halphén, G., Sur des suites de fractions analogues à la suite de Farey, Bulletin
de la Société Mathématique de France 5 (1877) 170–175.
[125] Hamilton, W. R., Memorandum respecting a new System of Roots of Unity,
The London, Edinburgh and Dublin Philosophical Magazine and Journal of
Science, 4th series, 12 (1856) 446.
[126] Hamilton, W. R., Account of the Icosian Calculus, Proceedings of the Royal
Irish Academy 6 (1858) 415–416.
[127] Harary, F., Graph Theory, Addison-Wesley, Reading MA, 1969.
[128] Harary, F., Palmer, E. M., Graphical enumeration, Academic Press, New
York NY, 1973.
[129] Harkin, D., On the mathematical work of François-Édouard-Anatole Lucas,
L’Enseignement Mathématique (2) 3 (1957) 276–288.
[130] Hartsfield, N., Ringel, G., Pearls in Graph Theory, Academic Press, Boston
MA, 1990.
Bibliography 305

[131] Hayes, B., Introducing a department concerned with the pleasures of com-
putation, Scientific American 249 (1983) 22–36.
[132] Hayes, P. J., A note on the Towers of Hanoi problem, The Computer Journal
20 (1977) 282–285.
[133] Haynes, T. W., Hedetniemi, S. T., Slater, P. J., Fundamentals of domination
in graphs, Marcel Dekker, New York NY, 1998.
[134] Heard, R. J., The Tower of Hanoi as a Trivial Problem, The Computer
Journal 27 (1984) 90.
[135] Hering, H., Zur Mathematisierung des »Turm von Hanoi«-Spiels, Der
mathematisch-naturwissenschaftliche Unterricht 26 (1973) 408–411.
[136] Hering, H., Minimalstrategien in endlichen Systemen, Der Mathematikun-
terricht 23 (1977) 25–35.
[137] Hering, H., Genetische Mathematisierung am Beispiel einer Variante des
Turm-von-Hanoi-Spiels, Didaktik der Mathematik 4 (1978) 307–317.
[138] Hering, H., Varianten zum “Turm von Hanoi-Spiel” – ein Weg zu einfachen
Funktionalgleichungen, Der Mathematikunterricht 25 (1979) 82–92.
[139] Heroor, V. D., The history of mathematics and mathematicians of India,
Vidya Bharathi Karnataka, Bangalore, 2006.
[140] Hinz, A. M., An Iterative Algorithm for the Tower of Hanoi with Four Pegs,
Computing 42 (1989) 133–140.
[141] Hinz, A. M., The Tower of Hanoi, L’Enseignement Mathématique (2) 35
(1989) 289–321.
[142] Hinz, A. M., Pascal’s Triangle and the Tower of Hanoi, The American Math-
ematical Monthly 99 (1992) 538–544.
[143] Hinz, A. M., Shortest Paths Between Regular States of the Tower of Hanoi,
Information Sciences 63 (1992) 173–181.
[144] Hinz, A. M., Square-free Tower of Hanoi sequences, L’Enseignement Mathé-
matique (2) 42 (1996) 257–264.
[145] Hinz, A. M., The Tower of Hanoi, in: K.-P. Shum, E. J. Taft, Z.-X. Wan
(Eds.), Algebras and Combinatorics, Springer, Singapore, 1999, 277–289.
[146] Hinz, A. M., A straightened proof for the uncountability of R, Elemente der
Mathematik 65 (2010) 26–28.
[147] Hinz, A. M., Tower of Hanoi, Nieuw Archief for Wiskunde (5) 12 (2011) 239.
[148] Hinz, A. M., Graph theory of tower tasks, Behavioural Neurology 25 (2012)
13–22.
306 Bibliography

[149] Hinz, A. M., Klavžar, S., Milutinović, U., Parisse, D., Petr, C., Metric prop-
erties of the Tower of Hanoi graphs and Stern’s diatomic sequence, European
Journal of Combinatorics 26 (2005) 693–708.
[150] Hinz, A. M., Klavžar, S., Zemljič, S. S., Sierpiński graphs as spanning sub-
graphs of Hanoi graphs, Central European Journal of Mathematics, to ap-
pear.
[151] Hinz, A. M., Kostov, A., Kneißl, F., Sürer, F., Danek, A., A mathematical
model and a computer tool for the Tower of Hanoi and Tower of London
puzzles, Information Sciences 179 (2009) 2934–2947.
[152] Hinz, A. M., Parisse, D., On the Planarity of Hanoi Graphs, Expositiones
Mathematicae 20 (2002) 263–268.
[153] Hinz, A. M., Parisse, D., Coloring Hanoi and Sierpiński graphs, Discrete
Mathematics 312 (2012) 1521–1535.
[154] Hinz, A. M., Parisse, D., The Average Eccentricity of Sierpiński Graphs,
Graphs and Combinatorics 28 (2012) 671–686.
[155] Hinz, A. M., Schief, A., The average distance on the Sierpiński gasket, Prob-
ability Theory and Related Fields 87 (1990) 129–138.
[156] Hoseiny Farahabady, M., Imani, N., Sarbazi-Azad, H., Some topological and
combinatorial properties of WK-recursive mesh and WK-pyramid intercon-
nection networks, Journal of Systems Architecture 54 (2008) 967–976.
[157] Hunter, W. S., Human behavior, The University of Chicago Press, Chicago
IL, 1928.
[158] Imrich, W., Klavžar, S., Rall, D. F., Topics in Graph Theory, A K Peters,
Wellesley MA, 2008.
[159] Jakovac, M., Klavžar, S., Vertex-, edge-, and total-colorings of Sierpiński-like
graphs, Discrete Mathematics 309 (2009) 1548–1556.
[160] Joyner, D., Adventures in Group Theory, Second edition, The Johns Hopkins
University Press, Baltimore MD, 2008.
[161] Kak, S., The Golden Mean and the Physics of Aesthetics, in: B. S. Yadav,
M. Mohan (Eds.), Ancient Indian Leaps into Mathematics, Birkhäuser, New
York NY, 2011, 111–119.
[162] Kaplan, R., The nothing that is, Penguin Books, London, 2000.
[163] Kauffman, L. H., Tangle complexity and the topology of the Chinese rings,
in: J. P. Mesirov, K. Schulten, D. W. Sumners (Eds.), Mathematical Ap-
proaches to Biomolecular Structure and Dynamics, Springer, New York NY,
1996, 1–10.
[164] Keister, W., Locking disc puzzle, US-Patent Nr. 3,637,215, 1972.
Bibliography 307

[165] Keister, W., Pattern-matching puzzle, US-Patent Nr. 3,637,216, 1972.


[166] Klahr, D., Goal formation, planning, and learning by pre-school problem
solvers or: “My socks are in the dryer”, in: R. S. Siegler (Ed.), Children’s
Thinking: What Develops?, Lawrence Erlbaum, Hillsdale NJ, 1978, 181–212.
[167] Klahr, D., Robinson, M., Formal assessment of problem-solving and planning
processes in preschool children, Cognitive Psychology 13 (1981) 113–148.
[168] Klavžar, S., Coloring Sierpiński graphs and Sierpiński gasket graphs, Tai-
wanese Journal of Mathematics 12 (2008) 513–522.
[169] Klavžar, S., Milutinović, U., Graphs S(n, k) and a variant of the Tower of
Hanoi problem, Czechoslovak Mathematical Journal 47(122) (1997) 95–104.
[170] Klavžar, S., Milutinović, U., Simple Explicit Formulas for the Frame-Stewart
Numbers, Annals of Combinatorics 6 (2002) 157–167.
[171] Klavžar, S., Milutinović, U., Petr, C., Combinatorics of topmost discs of
multi-peg Tower of Hanoi problem, Ars Combinatoria 59 (2001) 55–64.
[172] Klavžar, S., Milutinović, U., Petr, C., On the Frame-Stewart algorithm for
the multi-peg Tower of Hanoi problem, Discrete Applied Mathematics 120
(2002) 141–157.
[173] Klavžar, S., Milutinović, U., Petr, C., 1-perfect codes in Sierpiński graphs,
Bulletin of the Australian Mathematical Society 66 (2002) 369–384.
[174] Klavžar, S., Milutinović, U., Petr, C., Hanoi graphs and some classical num-
bers, Expositiones Mathematicae 23 (2005) 371–378.
[175] Klavžar, S., Mohar, B., Crossing Numbers of Sierpiński-Like Graphs, Journal
of Graph Theory 50 (2005) 186–198.
[176] Klein, C. S., Minsker, S., The super towers of Hanoi problem: large rings on
small rings, Discrete Mathematics 114 (1993) 283–295.
[177] Knuth, D., All Questions Answered, Notices of the American Mathematical
Society 49 (2002) 318–324.
[178] Knuth, D. E., Computer Science and Mathematics, American Scientist 61
(1973) 707–713.
[179] Knuth, D. E., The Art of Computer Programming. Vol. 1, Third edition,
Addison-Wesley, Reading MA, 1997.
[180] Knuth, D. E., The Art of Computer Programming. Vol. 4, Fasc. 2, Addison-
Wesley, Upper Saddle River NJ, 2005.
[181] Köhler, T., Überschneidungszahlen spezieller Hanoi- und Sierpiński-
Graphen, Diploma thesis, Ludwig-Maximilians-Universität, München, 2011.
308 Bibliography

[182] König, D., Theorie der endlichen und unendlichen Graphen, B. G. Teubner,
Leipzig, 1986.
[183] Korf, R. E., Best-First Frontier Search with Delayed Duplicate Detection,
in: Nineteenth National Conference on Artificial Intelligence (AAAI-2004),
The MIT Press, Cambridge MA, 2004, 650–657.
[184] Korf, R. E., Linear-Time Disk-Based Implicit Graph Search, Journal of the
ACM 55 (2008) Art. 26.
[185] Korf, R. E., Felner, A., Recent Progress in Heuristic Search: A Case Study of
the Four-Peg Towers of Hanoi Problem, in: M. M. Veloso (Ed.), Proceedings
of the Twentieth International Joint Conference on Artificial Intelligence
(IJCAI-07), AAAI Press, Menlo Park CA, 2007, 2324–2329.
[186] Kotovsky, K., Hayes, J. R., Simon, H. A., Why Are Some Problems Hard?
Evidence from Tower of Hanoi, Cognitive Psychology 17 (1985) 248–294.
[187] Kummer, E. E., Über die Ergänzungssätze zu den allgemeinen Reciproci-
tätsgesetzen, Journal für die reine und angewandte Mathematik 44 (1852)
93–146.
[188] Legendre, A. M., Essai sur la théorie des nombres, Seconde édition, Courcier,
Paris, 1808.
[189] Leibnitz, G. W., Explication de l’arithmétique binaire, Histoire de
l’Académie Royale des Sciences 1703, 85–89.
[190] Leiss, E. L., Solving the “Towers of Hanoi” on Graphs, Journal of Combina-
torics, Information & System Sciences 8 (1983) 81–89.
[191] Leiss, E. L., Finite Hanoi problems: how many discs can be handled?, Con-
gressus Numerantium 44 (1984) 221–229.
[192] Levy, H., Lessman, F., Finite Difference Equations, Macmillan, New York
NY, 1961.
[193] Li, C.-K., Nelson, I., Perfect codes on the Towers of Hanoi graph, Bulletin
of the Australian Mathematical Society 57 (1998) 367–376.
[194] Lin, C.-H., Liu, J.-J., Wang, Y.-L., Yen, W. C.-K., The Hub Number of
Sierpiński-Like Graphs, Theory of Computing Systems 49 (2011) 588–600.
[195] Lindley, E. H., A study of puzzles with special reference to the psychology
of mental adaption, American Journal of Psychology 8 (1897) 431–493.
[196] Lindquist, N. F., Solution to Problem 1350, Mathematics Magazine 64 (1991)
200.
[197] van Lint, J. H., Wilson, R. M., A Course in Combinatorics, Second edition,
Cambridge University Press, Cambridge, 2001.
Bibliography 309

[198] Lipscomb, S. L., A universal one-dimensional metric space, Lecture Notes in


Mathematics 378 (1974) 248–257.
[199] Lipscomb, S. L., On imbedding finite-dimensional metric spaces, Transac-
tions of the American Mathematical Society 211 (1975) 143–160.
[200] Lipscomb, S. L., Fractals and Universal Spaces in Dimension Theory,
Springer, New York NY, 2009.
[201] Lipscomb, S. L., The Quest for Universal Spaces in Dimension Theory, No-
tices of the American Mathematical Society 56 (2009) 1418–1424.
[202] de Longchamps, G., Variétés, Journal de Mathématiques Spéciales (2) 2
(1883) 286–287.
[203] Lovász, L., Random Walks on Graphs: A Survey, in: D. Miklós, V. T. Sós
and T. Szőnyi (Eds.), Combinatorics, Paul Erdős is eighty, Vol. 2, János
Bolyai Mathematical Society, Budapest, 1996, 353–397.
[204] Lu, X., Towers of Hanoi Graphs, International Journal of Computer Math-
ematics 19 (1986) 23–38.
[205] Lu, X.-M., Towers of Hanoi Problem with Arbitrary k ≥ 3 Pegs, International
Journal of Computer Mathematics 24 (1988) 39–54.
[206] Lucas, É., Sur la théorie des nombres premiers, Atti della Reale Accademia
delle Scienze di Torino 11 (1875-1876) 928–937.
[207] Lucas, É., Note sur l’application des séries récurrentes à la recherche de la
loi de distribution des nombres premiers, Comptes Rendus des Séances de
l’Académie des Sciences 82 (1876) 165–167.
[208] Lucas, É., Théorie des Fonctions Numérique Simplement Périodiques, Amer-
ican Journal of Mathematics 1 (1878) 184–240, 289–321.
[209] Lucas, É., Récréations Mathématiques, Gauthier-Villars et fils, Paris, 1882.
[210] Lucas, É., Jeux Scientifiques, Première Série, No 3, La Tour d’Hanoï, Cham-
bon & Baye/Édouard Lucas, Paris, 1889.
[211] Lucas, É., Nouveaux Jeux Scientifiques de M. Édouard Lucas, La Nature 17
(1889) 301–303.
[212] Lucas, É., Les appareils de calcul et les jeux de combinaisons, Revue Scien-
tifique de la France et de l’Étranger 45 (1890) 1–13.
[213] Lucas, É., Théorie des nombres, Tome premier, Gauthiers-Villars et fils,
Paris, 1891.
[214] Lucas, É., Récréations Mathématiques, Deuxième édition, Gauthier-Villars
et fils, Paris, 1891.
310 Bibliography

[215] Lucas, É., Récréations Mathématiques III, Gauthier-Villars et fils, Paris,


1893.
[216] Lucas, É., Récréations Mathématiques IV, Gauthier-Villars et fils, Paris,
1894.
[217] Lucas, É., L’arithmétique amusante, Gauthiers-Villars et fils, Paris, 1895.
[218] Lucas, É., Récréations Mathématiques II, Deuxième édition, Gauthier-
Villars et fils, Paris, 1896.
[219] Lucas, É., Notes sur M. Edouard Lucas Arithméticien, Yvert et Tellier,
Amiens, 1907.
[220] Lunnon, W. F., The Reve’s Puzzle, The Computer Journal 29 (1986) 478.
[221] Majumdar, A. A. K., A Note on the Iterative Algorithm for the Reve’s
Puzzle, The Computer Journal 37 (1994) 463–464.
[222] Majumdar, A. A. K., The generalized four-peg Tower of Hanoi problem,
Optimization 29 (1994) 349–360.
[223] Majumdar, A. A. K., The divide-and-conquer approach to the generalized
p-peg Tower of Hanoi problem, Optimization 34 (1995) 373–378.
[224] Majumdar, A. A. K., Halder, A., A Recursive Algorithm for the Bottleneck
Reve’s Puzzle, Sūrikaisekikenkyūsho Kōkyūroku 947 (1996) 150–161.
[225] Mallion, R. B., The Six (or Seven) Bridges of Kaliningrad: a Personal Eu-
lerian Walk, 2006, MATCH Communications in Mathematical and in Com-
puter Chemistry 58 (2007) 529–556.
[226] Mandelbrot, B., Les objets fractals: forme, hasard et dimension, Flammar-
ion, Paris, 1975.
[227] Mandelbrot, B. B., Fractals: form, chance, and dimension, W. H. Freeman,
San Francisco CA, 1977.
[228] Mandelbrot, B. B., The fractal geometry of nature, W. H. Freeman, New
York NY, 1982.
[229] Martin, G. E., Counting: The Art of Enumerative Combinatorics, Springer,
New York NY, 2001.
[230] Matsuura, A., Exact Analysis of the Recurrence Relations Generalized from
the Tower of Hanoi, SIAM Proceedings in Applied Mathematics 129 (2008)
228–233.
[231] Maurer, S. B., Ralston, A., Discrete Algorithmic Mathematics, Third edi-
tion, A K Peters, Wellesley MA, 2004.
[232] Meier, M., Exzentrizitäten in Hanoi-Graphen, Diploma thesis, Ludwig-
Maximilians-Universität, München, 2009.
Bibliography 311

[233] Meinhardt, H., The Algorithmic Beauty of Sea Shells, Springer, Berlin, 1995.
[234] Milutinović, U., Completeness of the Lipscomb universal space, Glasnik
Matematički, Serija III 27(47) (1992) 343–364.
[235] Minsker, S., The Towers of Hanoi Rainbow Problem: Coloring the Rings,
Journal of Algorithms 10 (1989) 1–19.
[236] Minsker, S., The Towers of Antwerpen problem, Information Processing Let-
ters 38 (1991) 107–111.
[237] Minsker, S., The Little Towers of Antwerpen problem, Information Process-
ing Letters 94 (2005) 197–201.
[238] Minsker, S., The Linear Twin Towers of Hanoi Problem, ACM SIGCSE
Bulletin 39 (2007) 37–40.
[239] Minsker, S., Another Brief Recursion Excursion to Hanoi, ACM SIGCSE
Bulletin 40 (2008) 35–37.
[240] Minsker, S., The Classical/Linear Hanoi Hybrid Problem: Regular Configu-
rations, ACM SIGCSE Bulletin 41 (2009) 57–61.
[241] Mohar, B., Thomassen, C., Graphs on Surfaces, The Johns Hopkins Univer-
sity Press, Baltimore MD, 2001.
[242] Montucla, J. F., Histoire des Mathématiques, Nouvelle édition, Tome pre-
mier, Henri Agasse, Paris, 1758.
[243] Neale, R., The temple of Hanoi—An old puzzler with a new twist, Games 6
(1982) 70.
[244] Newell, A., Simon, H. A., Human problem solving, Prentice-Hall, Englewood
Cliffs NJ, 1972.
[245] Noland, H., Problem 1350, Mathematics Magazine 63 (1990) 189.
[246] Nurlybaev, A. N., K zadaqe Lka o baxnh i ee obobwenii,
Izvesti Nacional~no Akademii Nauk Respubliki Kazahstan,
Seri fiziko-matematiqeska 182 (1995) 24–30.
[247] Obara, S., Hirayama, Y., The “Red and White Towers of Hanoi” Puzzle,
Journal of Hokkaido University of Education, Section II A 46 (1995) 61–69.
[248] Obara, S., Hirayama, Y., The “Distribution of the Tower of Hanoi” Puzzle,
Journal of Hokkaido University of Education, Section II A 47 (1997) 175–
179.
[249] Ozanam, J., Récréations mathématiques et physiques, Nouvelle édition,
Claude Jombert, Paris, 1735.
[250] Pacioli, L., De Viribus Quantitatis, Raccolta Vinciana, Milano, 1997.
312 Bibliography

[251] Pan, S., Richter, R. B., The Crossing Number of K11 Is 100, Journal of
Graph Theory 56 (2007) 128–134.
[252] Papin, P., Histoire de Hanoi, Fayard, Paris, 2001.
[253] Parisse, D., The Tower of Hanoi and the Stern-Brocot Array, Ph.D. thesis,
Ludwig-Maximilians-Universität, München, 1997.
[254] Parisse, D., On Some Metric Properties of the Sierpiński Graphs S(n, k),
Ars Combinatoria 90 (2009) 145–160.
[255] Park, S. E., The Group of Symmetries of the Tower of Hanoi Graph, The
American Mathematical Monthly 117 (2010) 353–360.
[256] de Parville, H., La tour d’Hanoï et la question du Tonkin, La Nature 12
(1884) 285–286.
[257] Peterson, J., Lanier, L. H., Studies in the comparative abilities of whites and
negroes, Williams and Wilkins, Baltimore MD, 1929.
[258] Petr, C., Kombinatorika posplošenih Hanojskih stolpov, Ph.D. thesis, Uni-
verza v Mariboru, Maribor, 2004.
[259] Pettorossi, A., Derivation of efficient programs for computing sequences of
actions, Theoretical Computer Science 53 (1987) 151–167.
[260] Piaget, J., La prise de conscience, Presses Universitaires de France, Paris,
1974.
[261] Poole, D., The Bottleneck Towers of Hanoi problem, Journal of Recreational
Mathematics 24 (1992) 203–207.
[262] Poole, D. G., Solution to Problem 1350, Mathematics Magazine 64 (1991)
200–201.
[263] Poole, D. G., The Towers and Triangles of Professor Claus (or, Pascal Knows
Hanoi), Mathematics Magazine 67 (1994) 323–344.
[264] Posamentier, A. S., Lehmann, I., The (Fabulous) Fibonacci Numbers,
Prometheus, Amherst NY, 2007.
[265] Pressman, I., Singmaster, D., The jealous husbands and The missionaries
and cannibals, The Mathematical Gazette 73 (1989) 73–81.
[266] Przytycki, J. H., Sikora, A. S., Topological insights from the Chinese Rings,
Proceedings of the American Mathematical Society 130 (2002) 893–902.
[267] Quint, J.-F., Harmonic analysis on the Pascal graph, Journal of Functional
Analysis 256 (2009) 3409–3460.
[268] Rankin, R. A., The first hundred years (1883–1983), Proceedings of the
Edinburgh Mathematical Society 26 (1983) 135–150.
Bibliography 313

[269] Reid, C. R., Sumpter, D. J. T., Beekman, M., Optimisation in a natural sys-
tem: Argentine ants solve the Towers of Hanoi, The Journal of Experimantal
Biology 214 (2011) 50–58.
[270] Richeson, D. S., Euler’s Gem, Princeton University Press, Princeton NJ,
2008.
[271] Ringel, G., Map color theorem, Springer, New York NY, 1974.
[272] Romik, D., Shortest paths in the Tower of Hanoi graph and finite automata,
SIAM Journal on Discrete Mathematics 20 (2006) 610–622.
[273] Roth, T., The Tower of Brahma revisited, Journal of Recreational Mathe-
matics 7 (1974) 116–119.
[274] Rukhin, A., On the Generalized Tower of Hanoi Problem: An Introduction
to Cluster Spaces, Master thesis, University of Maryland, College Park MD,
2004.
[275] Sachs, H., Stiebitz, M., Wilson, R. J., An Historical Note: Euler’s Königsberg
Letters, Journal of Graph Theory 12 (1988) 133–139.
[276] Sagan, H., Space-Filling Curves, Springer, New York NY, 1994.
[277] Saint-Cyr, J. A., Taylor, A. E., Lang, A. E., Procedural learning and neo-
striatal dysfunction in man, Brain 111 (1988) 941–959.
[278] Sainte-Laguë, A., Géométrie de situation et jeux, Gauthier-Villars, Paris,
1929.
[279] Sapir, A., The Tower of Hanoi with Forbidden Moves, The Computer Journal
47 (2004) 20–24.
[280] Sartorius v. Waltershausen, W., Gauss zum Gedächtnis, S. Hirzel, Leipzig,
1856.
[281] Scarioni, F., Speranza, H. G., A probabilistic analysis of an error-correcting
algorithm for the Towers of Hanoi puzzle, Information Processing Letters 18
(1984) 99–103.
[282] Scarioni, F., Speranza, M. G., The density function of the number of moves
to complete the Towers of Hanoi puzzle, Annals of Operations Research 1
(1984) 291–303.
[283] Schaal, S., Das Verhalten des Abstands perfekter Ecken und des
Durchmessers von Hanoi-Graphen Hpn für große n, Diploma thesis, Ludwig-
Maximilians-Universität, München, 2009.
[284] Scheinerman, E. R., Mathematics: A Discrete Introduction, Second edition,
Thomson Brooks/Cole, Belmont CA, 2006.
314 Bibliography

[285] Schmid, R. S., Überschneidungszahlen und Geschlecht bei Hanoi-


und Sierpiński-Graphen, Diploma thesis, Ludwig-Maximilians-Universität,
München, 2010.
[286] Schoute, P. H., De ringen van Brahma, Eigen Haard 10 (1884) 274–276,
286–287.
[287] Schröder, E., Ueber Algorithmen und Calculn, Archiv der Mathematik und
Physik, Zweite Reihe 5 (1887) 225–278.
[288] Schuh, F., Wonderlijke Problemen, Tweede druk, Thieme, Zutphen, 1943.
[289] Schützenberger, M., Une sotie sur les nombres parfaits, in: Les Nombres,
Pole, 2008, 58–61.
[290] Scorer, R. S., Grundy, P. M., Smith, C. A. B., Some binary games, Mathe-
matics Magazine 280 (1944) 96–103.
[291] Shallice, T., Specific impairments of planning, Philosophical Transactions of
the Royal Society of London Series B 298 (1982) 199–209.
[292] Sierpiński, W., Sur une courbe dont tout point est un point de ramification,
Comptes Rendus de l’Académie des Sciences Paris 160 (1915) 302–305.
[293] Sierpiński, W., Oeuvres choisies, PWN–Éditions Scientifiques de Pologne,
Warszawa, 1975.
[294] Simon, H. A., The Functional Equivalence of Problem Solving Skills, Cog-
nitive Psychology 7 (1975) 268–288.
[295] Singmaster, D., The Utility of Recreational Mathematics, in: R. K. Guy,
R. E. Woodrow (Eds.), The Lighter Side of Mathematics, The Mathematical
Association of America, Washington DC, 1994.
[296] Sloane, N. J. A., The On-Line Encyclopedia of Integer Sequences,
http://oeis.org, 2012–09–02.
[297] Soifer, A., The Mathematical Coloring Book, Springer, New York NY, 2009.
[298] Spencer, D. D., Game Playing with BASIC, Hayden Book, Rochelle Park
NJ, 1977.
[299] Stadel, M., Another nonrecursive algorithm for the Towers of Hanoi, ACM
SIGPLAN Notices 19 (1984) 34–36.
[300] Staples, E., The Tower of Hanoi problem with arbitrary start and end posi-
tions, SIGACT News 18 (1987) 61–64.
[301] Stern, M. A., Ueber eine zahlentheoretische Funktion, Journal für die reine
und angewandte Mathematik 55 (1858) 193–220.
[302] Stewart, B. M., Problems and solutions: Advanced problem 3918, The Amer-
ican Mathematical Monthly 46 (1939) 363.
Bibliography 315

[303] Stewart, B. M., Problems and solutions: Advanced problems: Solutions:


3918, The American Mathematical Monthly 48 (1941) 217–219.
[304] Stewart, I., Le lion, le lama et la laitue, Pour la Science 142 (1989) 102–107.
[305] Stewart, I., Another Fine Math You’ve Got Me Into. . ., W. H. Freeman and
Company, New York NY, 1992.
[306] Stewart, I., Four Encounters With Sierpiński’s Gasket, The Mathematical
Intelligencer 17 (1995) 52–64.
[307] Stewart, I., How to Cut a Cake, Oxford University Press, New York NY,
2006.
[308] Stewart, I., Sierpiński’s Pathological Curve and Its Modern Incarnations,
Wiadomości Matematyczne 48(2) (2012) 239–246.
[309] Stierstorfer, Q., Perfekte Codes auf Hanoi- und Sierpiński-Graphen, Diploma
thesis, Ludwig-Maximilians-Universität, München, 2007.
[310] Stirzaker, D., Stochastic Processes & Models, Oxford University Press, Ox-
ford, 2005.
[311] Stockmeyer, P. K., Variations on the four-post Tower of Hanoi puzzle, Con-
gressus Numerantium 102 (1994) 3–12.
[312] Stockmeyer, P. K., The Average Distance between Nodes in the Cyclic Tower
of Hanoi Digraph, in: Y. Alavi, D. R. Lick, A. Schwenk (Eds.), Combina-
torics, Graph Theory, and Algorithms, New Issues Press, Kalamazoo MI,
1999, 799–808.
[313] Stockmeyer, P. K., The Tower of Hanoi: A Bibliography, manuscript, 2005.
[314] Stockmeyer, P. K., The Tower of Hanoi for humans, manuscript, 2009.
[315] Stockmeyer, P. K., Bateman, C. D., Clark, J. W., Eyster, C. R., Harrison,
M. T., Loehr, N. A., Rodriguez, P. J., Simmons, J. R. III, Exchanging disks
in the Tower of Hanoi, International Journal of Computer Mathematics 59
(1995) 37–47.
[316] Stockmeyer, P. K., Lunnon, F., New variations on the Tower of Hanoi, Con-
gressus Numerantium 201 (2010) 277–287.
[317] Stone, R. G., Points recurring: The History of a Railway Problem, SIGPLAN
Notices 17 (1982) 88–94.
[318] Strichartz, R. S., Differential Equations on Fractals, Princeton University
Press, Princeton NJ, 2006.
[319] Strohhäcker, S., A program to find distances in Tower of Hanoi and related
graphs, Technical report, Technische Universität München, Garching, 2008.
316 Bibliography

[320] Šunić, Z., Twin Towers of Hanoi, European Journal of Combinatorics 33


(2012) 1691–1707.
[321] Swetz, F. J., Leibniz, the Yijing, and the Religious Conversion of the Chinese,
Mathematics Magazine 76 (2003) 276–291.
[322] Szegedy, M., In How Many Steps the k Peg Version of the Towers of Hanoi
Game Can Be Solved?, Lecture Notes in Computer Science 1563 (1999) 356–
361.
[323] Teufl, E., Wagner, S., Enumeration of matchings in families of self-similar
graphs, Discrete Applied Mathematics 158 (2010) 1524–1535.
[324] Teufl, E., Wagner, S., Resistance Scaling and the Number of Spanning Trees
in Self-Similar Lattices, Journal of Statistical Physics 142 (2011) 879–897.
[325] Tunstall, J. R., Improving the utility of the Tower of London, a neuropsy-
cholgical test of planning, M.Phil. thesis, Griffith University, Nathan QLD,
1999.
[326] Tutte, W. T., Graph Theory As I Have Known It, Clarendon Press, Oxford,
1998.
[327] Tutte, W. T., Graph Theory, Cambridge University Press, Cambridge, 2001.
[328] Urbiha, I., Some properties of a function studied by De Rham, Carlitz and
Dijkstra and its relation to the (Eisenstein-)Stern’s diatomic sequence, Math-
ematical Communications 6 (2001) 181–198.
[329] Vajda, S., Fibonacci & Lucas numbers, and the Golden Section, Ellis Hor-
wood, Chichester, 1989.
[330] Wallis, J., De Algebra Tractatus, Theatrum Sheldonianum, Oxford, 1693.
[331] Walsh, T. R., The Towers of Hanoi revisited: moving the rings by counting
the moves, Information Processing Letters 15 (1982) 64–67.
[332] Walsh, T. R., A case for iteration, Congressus Numerantium 40 (1983) 409–
417.
[333] Walsh, T. R., Iteration strikes back—at the Cyclic Towers of Hanoi, Infor-
mation Processing Letters 16 (1983) 91–93.
[334] Ward, G., Allport, A., Planning and problem-solving using the Five-disc
Tower of London task, The Quarterly Journal of Experimental Psychology
50A (1997) 49–78.
[335] Weil, A., Souvenirs d’apprentissage, Birkhäuser, Basel, 1991.
[336] West, D. B., Introduction to Graph Theory, Second edition, Prentice Hall,
Upper Saddle River NJ, 2001.
[337] Wexelblat, R. L., Editorial, ACM SIGPLAN Notices 20 (1985) 1.
Bibliography 317

[338] Wiesenberger, H. L., Stochastische Eigenschaften von Hanoi- und Sierpiński-


Graphen, Diploma thesis, Ludwig-Maximilians-Universität, München, 2010.
[339] Williams, H. C., Édouard Lucas and Primality Testing, John Wiley & Sons,
New York NY, 1998.
[340] Wilson, R. J., An Eulerian Trail Through Königsberg, Journal of Graph
Theory 10 (1986) 265–275.
[341] Wilson, R. J., Graph Theory, in: I. M. James (Ed.), History of Topology,
Elsevier, Amsterdam, 1999, 503–529.
[342] Wilson, R. J., Euler, lecture held at Gresham College, London, 2002.
[343] Wolfram, S., Geometry of binomial coefficients, The American Mathematical
Monthly 91 (1984) 566–571.
[344] Wolfram, S., A new kind of science, Wolfram Media, Champaign IL, 2002.
[345] Wood, D., The towers of Brahma and Hanoi revisited, Journal of Recre-
ational Mathematics 14 (1981/82) 17–24.
[346] Wood, D., Adjudicating a Towers of Hanoi Contest, International Journal
of Computer Mathematics 14 (1983) 199–207.
[347] Woodin, W. H., The tower of Hanoi, in: H. G. Dales, G. Oliveri (Eds.), Truth
in Mathematics, Oxford University Press, New York NY, 1998.
[348] Yap, H. P., Total Colourings of Graphs, Lecture Notes in Mathematics 1623,
Springer, Berlin, 1996.
[349] van Zanten, A. J., The complexity of an optimal algorithm for the Gener-
alized Tower of Hanoi problem, International Journal of Computer Mathe-
matics 36 (1990) 1–8.
[350] van Zanten, A. J., An Optimal Algorithm for the Twin-Tower Problem, Delft
Progress Report 15 (1991) 33–50.
[351] Zhang, Z., Wu, S., Comellas, F., The number of spanning trees of Hanoi
graphs, manuscript, 2011.
[352] Zhou Y., I Ching (Book of Changes), translated by James Legge, Bantam,
New York NY, 1969.
Name Index

Alcuin of York, 34 Chaucer, G., 46


Allardice, R. E., 43 Chen, G.-H., 159
Allouche, J.-P., 76, 78, 248, 250 Chen, H.-R., 159
Allport, A., 227 Chen, L.-C., 101
Arett, D., 192, 195 Chen, Xiao, 190
Astoorian, D., 76 Chen, Xiaomin, 225
Atkinson, M. D., 250 Claus (de Siam), N., 1, 3, 10, 12, 13,
Aumann, S., 188, 206 43
Azriel, D., 258, 259 Comellas, F., 101
Conway, J. H., 66
Ball, W. W. R., 1 Coxeter, H. S. M., 2
Barlow, M. T., 121 Crowe, D. W., 82
Behzad, M., 33 Culin, S., 4
Bell, E. T., 13 Cull, P., 45, 98, 160, 187
Bennish, J., 129
Berend, D., 249, 250, 255–258 D’Angeli, D., 127
Berg, W. K., 230 Décaillot, A.-M., 12, 13
Berger, T., 155 Della Vecchia, G., 159
Berlekamp, E. R., 66 Dinitz, Y., 225, 226
Binet, J. P. M., 265 Domoryad, A. P., 129
Bode, J.-P., 179 Donno, A., 127
Boettcher, S., 67 Dorée, S., 192, 195
Brendel, J., 101 Dress, F., 78
Brocot, L.-A., 18, 19 Dudeney, H. E., 4, 33, 46, 165, 170
Brousseau, Brother A., 187 Duh, D.-R., 159
Byrd, D. L., 230 Dunkel, O., 46, 167, 177, 190

Calkin, N., 124 Ecklund, E. F., 187


Cantor, G., 36 Ehler, C. L. G., 24
Cardano, G., 4, 41, 56 Eisenstein, G., 17
Cattin, A., 48 Emert, J. W., 255, 256
Chan, T.-H., 47, 119 Er, M. C., 45, 47, 80, 90, 94, 250
Chang, S.-C., 101 Euclid, 37
Chappelon, J., 172, 254 Euler, L., 23, 24, 26, 31, 267

A. M. Hinz et al., The Tower of Hanoi – Myths and Maths,


DOI: 10.1007/978-3-0348-0237-6, Ó Springer Basel 2013
320 Name Index

Faber, A. H., 230 Hirayama, Y., 92, 219


Fang, J. F., 159 Hoseiny Farahabady, M., 159
Felner, A., 167, 175, 179, 181, 182 Hunter, W. S., 48
Fermat, P., 9, 36
Fibonacci, L., 11, 14 Imani, N., 159
Finsterwalder, S., 285
Jacobsthal, E., 110
Fleury, 24, 267
Jakovac, M., 158
Frame, J. S., 46, 166, 167, 174–177,
Jordan, C., 31
184, 185, 188, 190, 203
Fraser, A. Y., 43 Kauffman, L. H., 66
Fu, J.-S, 159 Keister, W., 65
Fu Xi, 5, 6 Kirchhoff, G. R., 101
Fyodorov, M., 74 Klahr, D., 48
Klavžar, S., 49, 149, 155, 156, 158,
Gauss, J. F. C., ix, 35, 36
186, 187, 209, 259
Gedeon, T. D., 250
Klein, C. S., 132, 138, 140
Gerety, C., 45
Knuth, D., 49, 186
Germain, S., ix
Köhler, T., 158
Glaisher, J. W. L., 122, 276 Korf, R. E., 167, 175, 179, 181, 182
Gonçalves, B., 67 Kummer, E. E., 19
Götz, K. A. M., 202, 232
Gravier, S., 156 Lanier, L. M., 48
Gray, F., 58 Legendre, A. M., 61
Grigorchuk, R., 198 Leibniz, G., 5
Gros, L., 4, 41, 53, 56, 58, 61 Leiss, E. L., 244, 258
Grundy, P. M., 44, 94, 242, 256 Leonardo Pisano, 11
Guan, D.-J., 243 Li, C.-K., 98
Guclu, H., 67 Liang, W.-Y., 159
Guibert, O., 49 Liang, Z., 263
Guy, R. K., 32, 66 Lin, C.-H., 156
Lindley, E. H., 48
Halāyudha, 13, 19 Lindquist, N. F., 93
Halder, A., 225 Lipscomb, S. L., 161, 163
Halphén, G., 19 Liu, J.-J., 156
Hamilton, W. R., 26–28 de Longchamps, G., 43
Hayes, B., 80 Lu, X., 45, 47, 93, 187
Hayes, P. J., 46 Lucas, É., vii, ix, 3–6, 9, 12, 19, 20,
Heawood, P. J., 32 56, 61, 78, 79, 113, 131, 132,
Heffter, L., 32 211–213, 263
Hering, H., 49, 242, 250 Lunnon, F., 215, 217
Hierholzer, C., 24
Hinz, A. M., 47, 76, 120, 132, 140, Majumdar, A. A. K., 225
144, 158, 179, 195, 196, 209, Mandelbrot, B. B, 19, 21
232, 236 Marinoni, G., 23
Name Index 321

Matsuura, A., 172, 254 Sainte-Laguë, A., 63


Meier, M., 182, 202 Sanges, C., 159
Mersenne, M., 11, 12 Sapir, A., 245, 248–250, 255–257
Milutinović, U., 49, 149, 155, 186, Sarbazi-Azad, H., 159
187, 209, 259 Sartorius von Waltershausen, W., 35
Minsker, S., 132, 138, 140, 218, 219, Scarioni, F., 90
221 Schief, A., 120, 144
Möbius, A. F., 31 Schmid, R. S., 195
Mohar, B., 156 Schoute, P. H., 2, 43, 82, 129
Mollard, M., 156 Schuh, F., 2
Montucla, J. F., 11 Schützenberger, M., 12
Scorer, R. S., 44, 94, 242, 256
Neale, R., 221 Shallice, T., x, 48, 227
Nelson, I., 98 Shallit, J., 76
Nelson, R. B., 255, 256 Shen, J., 190
Newell, A., 49 Sierpiński, W., 21
Ng, K.-L., 159 Sikora, A. S., 67
Noland, H., 91 Simon, H. A., 48, 49
Nurlybaev, A. N., 164 Smith, C. A. B., 44, 94, 242, 256
Soifer, A., 33
Obara, S., 92, 219 Solomon, N., 259
Olive, R., 43, 74, 76, 78, 79 Solomon, S., 225, 226, 255, 259
Owens, F. W., 255, 256 Speranza, M. G., 90
Ozanam, J., 3 Spitzer, A., 236
Stern, M., 17, 19
Pacioli, L., 4, 41 Stewart, B. M., 46, 166, 167, 177,
Parisse, D., 123, 151, 158, 195, 196, 184, 188
209 Stewart, I., vii, 23, 119
Park, S. E., 197 Stierstorfer, Q., 195
de Parville, H., 1, 2, 9, 43 Stockmeyer, P. K., 84, 196, 214, 215,
Pascal, B., 13 217, 250–252, 254–256, 259
Perkins, E. A., 121 Strohhäcker, S., 179, 182
Peterson, J., 48 Sturm, C., 43
Petr, C., 155, 186, 209, 259 Šunić, Z., 127, 198
Piaget, J., 48 Szegedy, M., 189, 190
Poole, D. G., 93, 121, 222–225
Przytycki, J. H., 67 Teufl, E., 101
Tian, B., 225
Randall, J., 76 Tunstall, J. R., 231
Rasmussen, P., 41, 56, 60 Tutte, W. T., 101
Rémond de Montmort, P., 15
Ringel, G., 32 Urbiha, I., 123
Romik, D., 113, 124, 125, 127, 144,
147 Vizing, V. G., 33
322 Name Index

Wagner, S., 101


Wallis, J., 41, 54, 56
Walsh, T. R., 250
Wang, L., 225
Wang, Y.-L., 156
Ward, G., 227
Weil, A., 39
Wiesenberger, H. L., 60, 154
Wilf, H. S., 124
Wilson, R. J., 23
Wolfram, S., 19
Wood, D., 44, 45, 47, 218, 222
Woodin, W. H., 45, 263
Wu, S., 101

Yang, W.-S., 101


Yen, W. C.-K., 156
Youngs, J. W., 32

van Zanten, A. J., 91, 219, 221, 226


Zhang, W., 41, 56
Zhang, Z., 101
Zhu, X. Z., 56, 60
Zhuge Liang, 4
Subject Index

abelian square, 69 baguenaudier, 4, 58


absorbing Markov chain, 146 basic double task, 127
absorbing state, 146 benzenoid hydrocarbon, 117
accelerated Chinese rings (ACR), 56 best buffer (disc), 137
adjacency matrix, 101, 293 binary carry sequence, 61
admissible arrangement, 77 binary logarithm, 123
admissible state, 77 binary tree, 94, 293
algorithm Binet’s formula, 265
Fleury’s, 24 binomial theorem, 15
greedy, 33, 64 bipartite graph, 33
human, 113, 129 bit, 5, 293
Olive’s, 74 Black and white Tower of Hanoi
Allouche-Sapir conjecture, 262 (BWTH), 215
alphabet, 63 black sheep formula, 15
alternating group, 27 Bottleneck Tower of Hanoi (BTH),
ape, 155 222
Argentine ants, 50 boxer rule, 106, 153, 188
Arithmetical triangle, 13, 16 breadth-first search (BFS), 179, 267
arrangement, 50 bridge, 25, 293
admissible, 77 Brocot sequence, 18, 110
association, 36 Burnside’s lemma, 40, 100
atom, 17
automaton cardinality, 38
cellular, 19 careful brothers problem, 34
Gros code, 58 Catalan number, 259
P1-, 82 Catalan permutation, 49
Romik’s, 125, 146 Cayley table, 84
Romik’s H3 -to-S3 , 144 ceiling function, 14
automorphism group, 40, 99 cellular automaton, 19
average distance, 60 center, 130, 184, 293
average eccentricity, 59 central peg, 251
normalized, 112 characteristic polynomial, 104
Chinese rings (CR), 3, 53
ba gua 八卦, 5 accelerated, 56

A. M. Hinz et al., The Tower of Hanoi – Myths and Maths,


DOI: 10.1007/978-3-0348-0237-6, Ó Springer Basel 2013
324 Subject Index

Chinese rings graph, 55 Non-subtower, 262


Chinese rings network, 68 Rainbow, 262
Chinese string (CS), 66 Ring, 67, 261
chromatic index, 33, 195 Stockmeyer’s, 254, 262
chromatic number, 33, 158, 195, 293 Strong Frame-Stewart, 187, 262
circuit, 23, 293 Total coloring, 33, 197
eulerian, 23, 294 connected graph, 25, 58, 293
Class 1 graph, 196 connectivity, 97, 158, 194, 293
Class 2 graph, 196 countable, 38
clique, 150, 293 crossing number, 32, 157
p-, 150 cube
clique number, 33 n-, 69
closed trail, 23, 296 Rubik’s, 100
closed walk, 296 curve
code dragon, 62
Gray, 58, 63, 80, 90, 243, 271 generalized Sierpiński, 163
(p, n)-Gray, 243 triangular Sierpiński, 160
Gros, 58 cycle, 54, 293
perfect, 58, 69, 97–99, 155 directed, 244
reflected binary, 59 hamiltonian, 26, 69, 294
codeword, 69 moderately enhanced, 257
coloring cycle of permutation, 28
of edges, 33, 56, 294 cyclic permutation, 28, 293
total, 33, 296 Cyclic Tower of Hanoi, 244
vertex, 33, 296
combination, 14 degree, 23, 293
combinatorial number, 15 maximum, 33
complete bipartite graph, 33 minimum, 192
complete digraph, 242 degree balanced, 228
complete graph, 31, 293 delayed duplicate detection (DDD),
complexity class NP, 30 179
complexity class P, 26 derangement, 17, 294
complexity of graph, 101 Devil’s peg, 165
conjecture diameter, 56, 294
Allouche-Sapir, 262 digraph, 132, 294
Double task, 262 complete, 242
Frame-Stewart, 46, 167, 175, strong, 244
178, 262 directed cycle, 244
Graceful tree, 33 discrepancy, 222
Guy’s, 32, 158, 261 distance, 58
Köhler’s, 261 average, 60
Korf-Felner, 262 graph, 58, 294
LDM pattern, 262 distance layered graph, 203
Monotonicity, 262 divide-and-conquer, 46
Subject Index 325

divine rule, 2 Fleury’s algorithm, 24


dodecahedral graph, 27 floor function, 14
dominating set, 99 forest, 101, 294
domination number, 99, 155 formula
Domoryad’s task, 129 Binet’s, 265
double P2 problem, 127 black sheep, 15
Double task conjecture, 262 Euler’s polyhedral, 31
dragon curve, 62 Four-in-a-row Tower of Hanoi, 254
dual graph, 30 fractal, 23
Dunkel’s lemma, 178 Frame number, 166, 185
Frame-Stewart conjecture, 46, 167,
eccentricity, 59, 294 175, 178, 262
average, 59 Frame-Stewart number, 166, 170,
total, 59, 296 186, 187
edge chromatic number, 33, 158 Frame-Stewart-type strategy, 252
edge coloring, 33, 56, 294 frontier search, 179
equipotent, 38 function
equivalence, 37 ceiling, 14
equivalence class, equiset, 37 Euler’s phi, 19, 110
equivalent elements, 37 excess, 182
equivalent states, 183 floor, 14
Euler’s number, 17 ruler, 61
Euler’s phi function, 19, 110 fundamental matrix, 147
Euler’s polyhedral formula, 31 Fundamental theorem of recursion,
eulerian circuit, 23, 294 265
eulerian graph, 130, 294
eulerian trail, 23, 294 game
even permutation, 27 Icosian, 26
even vertex, 25 solitaire, 211
excess, 36 Tetralogic, 261
excess function, 182 Trilogic, 261
exchange number, 66 Game of encounter, 15
Exchanging discs TH (EDTH), 214 general Sierpiński graph, 149
exponential Tower of Hanoi, 257 generalized Frame-Stewart number,
extreme vertex, 144, 152 254
generalized Sierpiński curve, 163
factor set, 40 genus, genera, 32, 157
factorial, 50 geodesic, 106, 294
falling power, 209 Golden section, 12, 50
Fibonacci number, 12, 122, 123, 129 Graceful tree conjecture, 33
Fibonacci sequence, 11, 117 graph, 23, 294
fixed point, 17 bipartite, 33
fixed point set, 40 Chinese rings, 55
flat state, 48, 236, 281 Class 1, 196
326 Subject Index

Class 2, 196 group action, 39


complete, 31, 293 Guy’s conjecture, 32, 158, 261
complete bipartite, 33
connected, 25, 58, 293 hamiltonian cycle, 26, 69, 294
distance layered, 203 hamiltonian graph, 26
dodecahedral, 27 hamiltonian path, 26, 294
dual, 30 Hamming graph, 191, 194
eulerian, 130, 294 Handshaking lemma, 26, 192, 294
general Sierpiński, 149 Hanoi graph, 44, 45, 94, 179, 190, 259
hamiltonian, 26 Hanoi rainbow problem (THR), 221
Hamming, 191, 194 Hanoi Towers group, 127, 198
Hanoi, 44, 45, 94, 179, 190, 259 Hausdorff dimension, 23
iterated complete, 159 heat map, 181
London, 231, 232 height, 94, 294
mixed, 132, 295 height of peg, 231
Oxford, 232 hexagram, 5
pancyclic, 159 Holländischer Baum, 24
Pascal, 121 human algorithm, 113, 129
path, 54, 295 Hutchinson operator, 160
planar, 31, 156 hyperbinary representation, 123
regular, 28, 295
Schreier, 198 icosagonal, 27
semi-eulerian, 130, 295 Icosian calculus, 27
Sierpiński, 44, 49, 121 Icosian game, 26
Sierpiński with base 3, 141 Icosian group, 27
Sierpiński-like, 121 idle peg, 74, 89
Sisyphean Hanoi, 104, 120, 122 imperial ruler, 61
Sisyphean Sierpiński, 142 inclusion-exclusion principle, 16, 294
state, 44 independence number, 158, 294
toroidal, 235 index, 36
underlying, 234 chromatic, 33, 195
graph distance, 58, 294 Wiener, 59, 118
Gray code, 58, 63, 80, 90, 243, 271 infinite face, 31
(p, n)-, 243 intercalation, 17
greedy algorithm, 33, 64 irregular state, 131
Gros code, 58 isomorphic, 39
Gros code automaton, 58 isomorphy of graphs, 36
Gros sequence, 60, 61, 74, 271 isomorphy of groups, 39
group iso-problems, 100
alternating, 27 iterated complete graph, 159
automorphism, 40, 99 iterated function system, 160
Hanoi Towers, 127, 198 Iverson’s convention, 20
Icosian, 27
symmetric, 27 Jacobsthal number, 111, 123
Subject Index 327

Jacobsthal sequence, 56, 110 Matrix-Tree theorem, 101


Jordan curve theorem, 31 maximum degree, 33
measure, 23
Kekulé structure, 117 mediant, 18
Kirchhoff matrix, 101 mediation, 18
Köhler’s conjecture, 261 Mersenne number, 11
Königsberg bridges problem, 23 Mersenne sequence, 11
Korf phenomenon, 182 Meru-Prastāra, 13, 20
Korf-Felner conjecture, 262 minimum degree, 192
Kuratowski’s theorem, 33, 294 missionaries and cannibals problem
(MC), 268
largest disc move (LDM), 112, 206 mixed graph, 132, 295
latin square, 84 moderately enhanced cycle, 257
LDM pattern conjecture, 262 monotonicity condition, 185
legal move, 72 Monotonicity conjecture, 262
lemma Monsters and Globes, 50
Burnside’s, 40, 100 morphic sequence, 248
Dunkel’s, 178 most economical, 175
Handshaking, 26, 192, 294 Mount Meru, 13
length, 29, 294 move
Linear Tower of Hanoi, 242, 251, 255 largest disc, 112, 206
Linear Tower of Hanoi on p pegs, 251 legal, 72
Linear Twin Hanoi, 221 type 0, 53
Lipscomb space, 161 type 1, 53
Little Tower of Antwerpen (LTA), multigraph, 23, 295
219
Locking disc puzzle, 65 natural number, 11, 295
London graph, 231, 232 network
loop, 268, 294 Chinese rings, 68
Lucas correspondence, 121 Tower of Hanoi, 67
Lucas number, 12, 111 WK-recursive, 159
Lucas sequence, 12 Noland’s problem, 93, 220
Lucas’s second problem, 131, 235, non-repetitive sequence, 63
262 Non-subtower conjecture, 262
normalized average eccentricity, 112
Markov chain, 146 NP-complete, 30
absorbing, 146 number
matching, 101, 294 Catalan, 259
perfect, 130, 295 chromatic, 33, 158, 195, 293
matrix clique, 33
adjacency, 101, 293 combinatorial, 15
fundamental, 147 crossing, 32, 157
Kirchhoff, 101 domination, 99, 155
transition, 146 edge chromatic, 33, 158
328 Subject Index

Euler’s, 17 hamiltonian, 26, 294


exchange, 66 Pattern-matching puzzle, 65
Fibonacci, 12, 122, 123, 129 peg
Frame, 166, 185 central, 251
Frame-Stewart, 166, 170, 186, Devil’s, 165
187 idle, 74, 89
generalized Frame-Stewart, 254 special, 137
independence, 158, 294 pendant vertex, 54, 295
Jacobsthal, 111, 123 perfect code, 58, 69, 97–99, 155
Lucas, 12, 111 perfect matching, 130, 295
Mersenne, 11 perfect state, 71, 165
natural, 11, 295 period-doubling sequence, 79
partition, 167 Peripheral phenomenon, 184
3-smooth, 172, 252 periphery, 182, 184, 275, 295
Stern, 123 permutation, 50, 295
Stirling of the second kind, 192, Catalan, 49
296 cyclic, 28, 293
tetrahedral, 46, 187 even, 27
total chromatic, 33, 158, 196 phenomenon
triangular, 35, 170 Korf, 182
Peripheral, 184
odd vertex, 25 pigeonhole principle, 50, 112, 295
Olive sequence, 76, 78, 104 p-in-a-row Tower of Hanoi, 251
Olive’s algorithm, 74 pit, 151, 295
Olympic Tower of London, 232 planar graph, 31, 156
orbit, 40 presumed minimal solution, 47, 167
Orbit-Stabilizer theorem, 40, 99 principle
order, 32, 295 inclusion-exclusion, 16, 294
Oxford graph, 232 pigeonhole, 50, 112, 295
problem
P1-automaton, 82 careful brothers, 34
P2 decision problem, 113, 145 double P2, 127
pancyclic graph, 159 Hanoi rainbow, 221
paper-folding sequence, 62 Königsberg bridges, 23
partition, 37, 295 Lucas’s second, 131, 235, 262
h-, 238 missionaries and cannibals, 268
strict, 38 Noland’s, 93, 220
trivial, 38 P2 decision, 113, 145
partition number, 167 rabbit, 11
Pascal graph, 121 river crossing, 34
Pascal’s triangle, 121 Triple-Tower, 221
path graph, 54, 295 Twin-Tower, 219
path in digraph, 134, 295 type P0, 71, 242
path in graph, 295 type P1, 82, 86, 212
Subject Index 329

type P2, 105, 212 breadth-first, 179, 267


type P3, 132 frontier, 179
type P4, 132 seed, 17
water, gas and electricity, 33 self-similar, 23
wolf, goat and cabbage, 34 semi-eulerian graph, 130, 295
problem isomorph, 50, 65 semi-perfect state, 134
prosody, 13 sequence
Prouhet-Thue-Morse sequence, 79, binary carry, 61
105 Brocot, 18, 110
puzzle Fibonacci, 11, 117
Locking disc, 65 Gros, 60, 61, 74, 271
Pattern-matching, 65 Jacobsthal, 56, 110
The Reve’s, 46, 165, 251 Lucas, 12
Mersenne, 11
quasigroup, 84 morphic, 248
Steiner, 84 non-repetitive, 63
quaternary, 165 Olive, 76, 78, 104
quotient set, 37 paper-folding, 62
period-doubling, 79
rabbit problem, 11 Prouhet-Thue-Morse, 79, 105
radius, 130, 295 3-smooth, 172, 252
Rainbow conjecture, 262 square-free, 63, 75
recurrence, 11 Stern’s diatomic, 18, 115, 121
recursive, 10 strongly square-free, 69, 76
reflected binary code, 59 Thue, 79, 105
regular graph, 28, 295 set
regular state, 71, 165 dominating, 99
regularization, 132 factor, 40
representative, 38 fixed point, 40
right direction, 82 quotient, 37
Ring conjecture, 67, 261 Sierpiński curve
river crossing problem, 34 generalized, 163
Romik’s automaton, 125, 146 triangular, 160
Romik’s H3 -to-S3 automaton, 144 Sierpiński gasket, 21
root, 94, 179, 267, 295 Sierpiński graph, 44, 49, 121
rooted tree, 295 Sisyphean, 142
Rubik’s cube, 100 Sierpiński graph with base 3, 141
rule Sierpiński triangle (ST), 23, 101, 119,
boxer, 106, 153, 188 160
divine, 2 Sierpiński-like graph, 121
ruler function, 61 sign manual of Muhammad, 25
singleton, 294, 296
Schreier graph, 198 Sinner’s Tower of Hanoi, 225
search Sisyphean Hanoi graph, 104, 120, 122
330 Subject Index

Sisyphean Sierpiński graph, 142 Strong Frame-Stewart conjecture,


size of graph, 25, 296 187, 262
1-skeleton, 27, 296 strongly square-free sequence, 69, 76
3-smooth number, 172, 252 sub-exponential Tower of Hanoi, 257
3-smooth sequence, 172, 252 subdiagonal, 122
solitaire game, 211 subdivision, 33, 296
solution subfactorial, 17, 51
presumed minimal, 167 subtower, 168, 172
subtower, 175 subtower solution, 175
solvable double P2 task, 127 suffix, 127, 296
solvable Tower of Hanoi, 242 superdisc, 172, 209
spanning subgraph, 101, 191, 296 Switching Tower of Hanoi (STH), 49,
spanning tree, 101, 267 145, 149
special case, 135 symmetric group, 27
special peg, 137
Taegeukgi, 5
spreadsheet, 80
task
square, 64
basic double, 127
abelian, 69
Domoryad’s, 129
latin, 84
solvable double P2, 127
square-free sequence, 63, 75
tetrahedral number, 46, 187
stabilizer, 40
Tetralogic game, 261
standard, 37 The Reve’s puzzle, 46, 165, 251
standard case, 135 theorem
Star Tower of Hanoi, 251 binomial, 15
state, 131 Fundamental of recursion, 265
absorbing, 146 Jordan curve, 31
admissible, 77 Kuratowski’s, 33, 294
flat, 48, 236, 281 Matrix-Tree, 101
irregular, 131 Orbit-Stabilizer, 40, 99
perfect, 71, 165 Vizing’s, 33, 196
regular, 71, 165 Three-in-a-row TH, 221, 242
semi-perfect, 134 Thue sequence, 79, 105
state graph, 44 tit, 85, 296
Steiner quasigroup, 84 Toeplitz transform, 78
Stern number, 123 toroidal graph, 235
Stern’s diatomic array, 17, 110 total chromatic number, 33, 158, 196
Stern’s diatomic sequence, 18, 115, total coloring, 33, 296
121 Total coloring conjecture, 33, 197
Stirling number of the second kind, total eccentricity, 59, 296
192, 296 Tower of Antwerpen (TA), 218
Stockmeyer’s conjecture, 254, 262 Little, 219
strict partition, 38 Tower of Brahma, 1
strong digraph, 244 Tower of Hanoi (TH), vii, 2, 71
Subject Index 331

Black and white, 215 type


Bottleneck, 222 0 move, 53
Cyclic, 244 1 move, 53
Exchanging discs, 214 P0 problem, 71, 242
exponential, 257 P1 problem, 82, 86, 212
Four-in-a-row, 254 P2 problem, 105, 212
Linear, 242, 251, 255 P3 problem, 132
Linear on p pegs, 251 P4 problem, 132
p-in-a-row, 251
Sinner’s, 225 underlying graph, 234
solvable, 242
Star, 251 vertex
sub-exponential, 257 even, 25
Switching, 49, 145, 149 extreme, 144, 152
Three-in-a-row, 221, 242 odd, 25
weakly solvable, 258 pendant, 54, 295
Tower of Hanoi network, 67 vertex coloring, 33, 296
Tower of London (TL), 49, 227 Vizing’s theorem, 33, 196
Olympic, 232
walk, 28, 296
Tunstall’s, 232
closed, 296
Tower of Oxford, 227
water, gas and electricity problem, 33
Tower of Toronto, 50
weakly solvable Tower of Hanoi, 258
trail, 23, 296
Wiener index, 59, 118
closed, 23, 296
WK-recursive network, 159
eulerian, 23, 294
wolf, goat and cabbage problem, 34
transition matrix, 146
transitive, 37 yang 阳, 5
transposition, 27, 296 yi jing 易经, 5
tree, 94, 296 yin 阴, 5
binary, 94, 293
rooted, 295
spanning, 101, 267
triangle
Arithmetical, 13, 16
Pascal’s, 121
Sierpiński, 23, 101, 119, 160
triangular number, 35, 170
triangular Sierpiński curve, 160
trigram, 5
Trilogic game, 261
Triple-Tower problem (TTT), 221
trivial partition, 38
Tunstall’s Tower of London, 232
Twin-Tower problem (TT), 219
Symbol Index

α(n) – the first vertex of Rn , 54 BTHt – Bottleneck TH with discrep-


α(G) – independence number of ancy t, 222
graph G, 158
β – Brocot sequence, 18 C(G) – center of graph G, 183
γ(G) – domination number of graph Ð

C p – directed cycle on n vertices, 244
G, 99 cr(G) – crossing number of graph G,
δ(G) – minimum degree of G, 192 32
∆(G) – maximum degree of G, 33
∆ν – triangular numbers, 35 d(s, t) – distance between states s
ε(v) – eccentricity of vertex v, 59 and t, 88
κ(G) – connectivity of graph G, 97 d(s; j, k) = d(s, j n ) − d(s, k n ), 106
Σ(3) – triangular Sierpiński curve, d(G) – average distance on graph G,
160 60
τ (G) – complexity of graph G, 101 deg(v) – degree of vertex v, 24
ϕ – Euler’s phi function, 19 diam(G) – diameter of graph G, 58
χ(G) – chromatic number of graph
G, 33 E(G) – total eccentricity of graph G,
χ′ (G) – chromatic index of graph G, 59
33 ex(n) = ε(0n )−d(0n , 3n ) – the excess
χ′′ (G) – total chromatic number of function, 182
graph G, 33 EX(n) = diam(H4n ) − d(0n , 3n ), 182
ω(G) – clique number of graph G, 33
ω (n) – the last vertex of Rn , 54 f – paper-folding sequence, 63
n n
F 4 (= F S 4 ) – Frame numbers for 4
Altn – alternating group on n ele- pegs, 166, 174
ments, 27 Fk – Fibonacci numbers, 12
n
AT – Arithmetical triangle, 13 F p – Frame numbers for p pegs, 185
Aut(G) – set/group of automor- F S3n (= F3n ) – Frame-Stewart num-
phisms of a graph G, 40 bers for 3 pegs, 166
F S4n (= F4n ) – Frame-Stewart num-
B – set {0, 1}, 54 bers for 4 pegs, 166
b – Stern’s diatomic sequence, 115 F Spn – Frame-Stewart numbers for p
B(N ) = ∑N ν=0 b(ν), 116
−1
pegs, 186

A. M. Hinz et al., The Tower of Hanoi – Myths and Maths,


DOI: 10.1007/978-3-0348-0237-6, Ó Springer Basel 2013
334 Symbol Index

g – Gros sequence, 60 P4 – problem irregular to regular,


g(G) – genus of graph G, 32 132
g – binary carry sequence, 61
̃ Pk – path graph on k vertices, 59

H3n – Hanoi graphs on 3 pegs, 94 Q – set {0, 1, 2, 3}, 165


Ð
→ q(n) – sum of bits of n, 61
H n3 – Hanoi digraphs on 3 pegs, 132
H3∞ – Sisyphean Hanoi graph, 104
rad(G) – radius of graph G, 130
H4n – Hanoi graph on 4 pegs, 179
Rn – state graph of the Chinese rings,
Hpn – Hanoi graph on p pegs, 190
54
H (p) – Hanoi Towers group, 198
s(3) – 3-smooth sequence, 172
Jk – Jacobsthal numbers, 110 S3n – Sierpiński graph (with base 3),
141
Kp – complete graph on p vertices, Spn – general Sierpiński graph, 149
31 ST – Sierpiński triangle, 23


K p – complete digraph on p vertices, ST4 – digraph of the Star Tower of
242 Hanoi, 251
K(G) – Kirchhoff matrix of graph G, ST4n – Star Tower of Hanoi numbers,
101 252
Symn – symmetric group on n ele-
lb(x) – binary logarithm of x, 123
ments, 27
L – state graph of the classical Tower
Sym(T ) – symmetric group on T , 99
of London, 228
L3231 – Olympic Tower of London, 232 T – set {0, 1, 2}, 71
L4234 – state graph of Tunstall’s Tn - set of states with n discs, 132
Tower of London, 232 TH(D) – TH with oriented disc
Lnh – London graph, 232 moves corresponding to di-
Lk – Lucas numbers, 12 graph D, 242
←→
L 3 – digraph of the linear TH, 242
W (G) – Wiener index of graph G, 59
Mk – Mersenne numbers, 11
mod – the modulo operation, 20 Z – set {. . . , −2, −1, 0, 1, 2, . . .}, 67

N – set {1, 2, 3, . . .}, 11 zn (µ) – number of vertices s ∈ H3n


N0 – set {0, 1, 2, . . .}, 10 with d(s; k, j) = µ, 108
X g – fixed point set of g, 39
o – Olive sequence, 76 3 = 2/3, 85
Opn – Oxford graph, 232 ⊍ – union of pairwise disjoint sets, 72
A×A˙ = {(a, b) ∈ A × A ∣ a =/ b}, 100
P (G) – periphery of graph G, 184 ∣M ∣ – size of set M , 50
P0 – problem perfect to perfect, 71 ∣G∣ – order of graph G, 32
P1 – problem regular to perfect, 82 ∣∣G∣∣ – size of graph G, 25
P2 – problem regular to regular, 105 ∣∣∣G∣∣∣ – number of faces of planar
P3 – problem irregular to perfect, 132 graph G, 32
Symbol Index 335

(S) – binary truth value of state-


ment S, 20
(k`) – combinatorial number, 14
(K` ) – set of all subsets of K of order
`, 15
{ nk } – Stirling numbers of the second
kind, 199, 209, 296
bk – constant word, 54
nk – falling power, 209
M N – set of mappings from set N to
set M , 265
x ≈ y – x equivalent y, 37
[x] – equivalence class (equiset) of x,
37
⌊x⌋ – floor of x, 14
⌈x⌉ – ceiling of x, 14
X/Γ – factor set, 40
G ≅ G′ – isomorphic graphs, 39
k! – k factorial, 50
k ¡ – k subfactorial, 51
[k] – set {1, . . . , k}, 50
[k]0 – set {0, . . . , k − 1}, 50
V /E = V / ≈ – quotient set, 37
Γx – stabilizer of x, 39
Γ.x – orbit of x, 40
a
b
⊕ dc – mediant of ab and dc , 18
i j – a binary operation on T , 82

s j – best first move vector, 93


f ↾ D – function f restricted to do-


main D, 266

You might also like